SlideShare a Scribd company logo
1 of 126
Hematology
Archer’s Online USMLE Reviews
www.CcsWorkshop.com
All rights reserved
Anticoagulants







Heparin
Low Molecular Weight Heparin
Warfarin
Direct thrombin inhibitors –
leuperidin, argatroban
Factor Xa Inhibitors - fondaparinaux
Heparins
Antithrombin 3 is a naturally occuring slow inhibitor of clotting
pathway. Heparin binds to AT-3 and converts it from slow
to rapid inhibitor by forming a ternary complex with AT-3
and thrombin






Unfractionated Heparin : Unfractionated heparin is a
heterogeneous mixture of polysaccharide chains with a
mean molecular weight of 15,000 Daltons. Given I.V.
Requires Inpatient Rx. Higher incidence of Throbocytopenia
and osteopenia. Monitored by measuring APTT
LMWH: Derived from heparin. Molecular weight-5000 D.
Can be given twice daily doses on OP basis. Less incidence
of throbocytopenia and osteopenia. LMWHs have more anti
factor X a activity than UF heparin. LMWH activity is
monitored by factor Xa activity.
Heparins can cause Hyperkalemia
Heparin Induced Skin Necrosis














Affects middle age women with history of thrombotic
disease
Characterized by the formation of one or more painful red
plaques or necrotic skin lesions.
5 days or more after starting heparin treatment. Earlier in
those treated previously with heparin.
Some patients develop thrombocytopenia when lesion first
appears, often with paradoxical thrombosis.(HIT)  Is not
always associated with thrombocytopenia
A rare complication of heparin characterized by immune
complex formation and thrombosis.
HIT Should be suspected if the patient develop skin
necrosis in areas of SQ injection or IV site.
Rx – stop Heparin + start Direct thrombin inhibitors or
heparinoids ( Leupiridin or Argatroban )  ( i.e; treat like
HIT)
Warfarin
Mechanism of action
Antagonist
Skin necrosis
Uses
INR monitoring
Dealing with Supratherapeutic INR on case by case
basis
Dealing with Sub therapeutic INR ( APLA, OBESITY,
WARFARIN RESISTANCE, INTERACTIONS, NONCOMPLIANCE)

WARFARIN: MECHANISM OF ACTION
Vitamin K epoxide
WARFARIN

Vitamin K reduced
Inactive factors II,
VII, IX, and X
Proteins S and C

Active factors II,
VII, IX, and X
Proteins S and C

Prevents the reduction of vitamin K, which is essential for
activation of certain factors
Has no effect on previously formed thrombus
PLASMA HALF-LIVES OF VITAMIN KDEPENDENT PROTEINS
Factor II
Factor VII

72h
6h

Factor IX

24h

Factor X

36h

Peak anticoagulant effect may be delayed by 72 to 96 hours
PROTEIN S HALF LIFE : shorter than above
Coumadin Skin Necrosis
•

•
•

Warfarin Procoagulant Effect in
first few hours can cause
warfarin skin necrosis due to
thrombosis.
Concomitant heparin use can
decrease the incidence
Patients developing coumadin
skin necrosis should be
evaluated for Protein C
deficiency
Rx



Stop Coumadin
Use heparin until the necrotic lesions
heal.
Correcting SupraTherapeutic INR
Approach!












Warfarin has a narrow therapeutic index
Therapeutic INR typically targeted at 2-3.
The risk of bleeding increases significantly when the INR >
4-6. However, the absolute risk is still low at 5.5 bleeding
events per 1000 per day
Therefore, patients with an INR < 9 and no significant
bleeding  Manage by omitting subsequent doses of
warfarin, more frequent monitoring of the INR, and
resumption of therapy at a lower dose when the INR is
therapeutic.
When rapid reversal of the INR is needed, fresh frozen
plasma, prothrombin complex, or recombinant factor VIIa
can be administered.
Administration of coagulation factors provides only a
temporary solution due to the short half-life of the provided
clotting factors (3-4 hours for Factor VII), compared with a
duration of action of 2 to 5 days for warfarin, as well as
relative instability of clotting factors upon administration.
Administration of either fresh frozen plasma or factor
concentrates will decrease the PT/INR for 4 to 6 hours 
So, complete return to a therapeutic INR will require
supplementation with vitamin K
Rxng Supratherapeutic INR










Treatment of a supra therapeutic INR requires a
balance between reducing the risk for hemorrhage
while minimizing the risk of thrombembolism.
Treatment approaches are based on the current
INR, presence of bleeding, and the time frame in
which reversal is required.
Vitamin K1 CAN BE GIVEN to reverse the
anticoagulation effect of warfarin.
The most appropriate dose of vitamin K1 is
the one that lowers the INR to a safe level without
resulting in a subtherapeutic INR.
High doses of vitamin K1 are effective but may
lead to warfarin resistance for a week or more,
resulting in an increased risk for
thromboembolism. In such cases, heparin should
be given until the effects of the vitamin K1 are
complete. ( note this point)
Forms of Vitamin K





Available in subcutaneous, IV and oral forms
Subcutaneous route  delayed onset and is less
predictable.
If rapid reversal is desired, the IV route should be utilized,
as this route has the fastest onset of action. ( Historically,

intravenous vitamin K1 has been associated with an increased risk of anaphylaxis. A
retrospective review of anaphylactic reactions associated with IV vitamin K1 from the
Mayo Clinic revealed that the risk of anaphylaxis with vitamin K1 was 3 per 10,000
doses—a rate comparable to all forms of penicillin and less than that of IV iron
dextran. If is administered by the IV route, lower doses and slower infusion rates are
recommended)




Unless rapid reversal of the INR is critical, oral vitamin K1
is the preferred route of administration.
In the United States, oral vitamin K1 is only available as a
5 mg tablet (Mephyton®). Therefore, oral doses prescribed
should reflect even divisions of 5 mg (e.g., 2.5 mg).
Forms of Vitamin K

Route

Advantages

Disadvantages

IV

•

Fastest Onset of
Action

•

Subcutaneous

•

Lower risk of
anaphylaxis

•

Oral

•

Safest route
•Low risk of
anaphylaxis
•No IV site needed

•

Must be given by slow
IV infusion
•Warfarin resistance
Delayed onset
•Unpredictable
response
•Least desired route
Slower onset of action
•Warfarin resistance
Management of Elevated INR in Patients Receiving Vitamin K Antagonists
INR

Bleeding Present

Rapid Reduction Required

Management*

<5

No

No

Lower dose by 10% or
omit dose; resume at
lower dose when INR is
therapeutic i.e; 2 TO 3

5-9

No

No

Omit one or two doses,
resume at lower dose by
10%.

No

Yes — high risk

May give vitamin K 2.5 mg
orally if at increased risk
for bleeding.

No

Yes—surgery

2-4 mg oral vitamin K1 for
reduction of INR in 24
hours; if INR still high, can
repeat with 1-2 mg orally.

≥9

No

No

Hold dose and give
vitamin K1 10 mg orally to
reduce INR in 24 hours;
may repeat vitamin K1as
necessary. Resume at
lower dose when
therapeutic.

Any

Yes—Serious Bleeding

Yes

Hold dose, give vitamin K1
10 mg by slow IV infusion,
along with fresh frozen
plasma, prothrombin
complex, or recombinant
factor VIIa; vitamin K1 may
be repeated every 12
hours.

Any

Yes—Life Threatening

Yes

Hold dose, give
prothrombin complex
along with vitamin K1 10
mg by slow IV infusion;
repeat if necessary
depending on INR.






When the INR is elevated in a patient who
has been on a consistent dose of warfarin
for past few weeks  Always, consider
why it is elevated
Antibiotics can potentate by causing
Vitamin k deficiency.
If drug interaction is considered  stop
the other drug if possible. If stopping the
interacting drug is feasible and if INR < 5,
no need to reduce the warfarin dose
Addressing Sub
Therapeutic INR
Approach on increasing the
dose
Warfarin Resistance


Sub-Therapeutic INR






INR< 1.5 – Increase the dose by 10 to
20%, consider extra dose and repeat
INR in 4 to 8 days.
1.5 to 2.4 – Increase dose by 5 to
10%. Repeat INR in one to two weeks
2.5 to 3.5 – No change in dose.
Repeat INR is the # of consecutive inrange INRs (For example, if a patient
has had three consecutive in-range
INR values, recheck in 3 weeks.
Maximum repeatable period not
longer than 4 weeks )
Foods that can cause
sub-therapeutic INR


Foods with very high vitamin K content
(more than 200 mcg) — Brussel sprouts,
chickpeas, collard greens, coriander,
endive, kale, liver, parsley, red leaf
lettuce, spinach, Swiss chard, black or
green teas, turnip greens, watercress.
Foods with high vitamin K content
(100-200 mcg) include basil, broccoli,
butterhead lettuce, canola oil, chives,
coleslaw, cucumbers (with peel), green
onions, mustard greens, soybean oil.
Scenarios








DVT
Pulmonary Embolism
DVT prophylaxis
Atrial fibrillation
Prosthetic Valves
Mural thrombus
Acute Coronary Syndrome
Deep Vein Thrombosis
Pulmonary Embolism
Abnormal
Blood Flow

Abnormal
Vessel Wall

Abnormal
Blood

The Hypercoagulable State
Dr. Rudolph Virchow
1821-1902
Deep Vein Thrombosis
Clinical associations
Immobility
Obesity
Smoking
Cancer
Pregnancy
Estrogen therapy
Economy class syndrome






Economy-class syndrome is a name used
by the media to describe Venous
Thrombo-Embolism that can occur in
anyone - but has been of particular worry
in air travellers.
Incidence of DVT & PE increases with the
flight distance
Prevention tips : Drink fluids, avoid
caffeine and alcohol, keep moving legs
and wear graduated support stockings
designed for travel.
Proximal

Distal
Proximal vs. Distal DVT










Thrombosis confined to deep calf veins is Distal
DVT where as thrombosis at or above popliteal
veins is Proximal DVT.
A distal DVT becomes clinically important when it
extends proximally
Risk of PE is much higher with proximal DVT.
Silent PE occurs in about 50% of pts with
proximal DVT.
All patients with Proximal DVT are at increased
long term risk for chronic venous insufficiency.
Compression ultrasound is more sensitive to
proximal DVT when compared to distal DVT.
Idiopathic DVT






A case of DVT where no evidence of
underlying obvious cause such as
surgery, trauma or known
malignancy is present.
Search for a hypercoagulable state in
such conditions
Hypercoagulable states : inherited or
acquired
Evaluating Idiopathic
Venous
Thromboembolism
APPROACH
Hypercoagulabilty – Whom to test?







DVT occurring in a pt < 50 yrs of age
Positive Family Hx of Venous
thromboembolism
Recurrent episodes of unexplained Venous
thromboembolism
Don’t forget to include Homocysteine When homocysteine levels are elevated in
the presence of factor V Leiden or the
prothrombin gene G20210A mutation, risk
of recurrent thrombosis appears to be
increased beyond the risk associated with
any one defect alone
Hypercoagulabilty – When to test?




Heparin
• Controversial AT-III (heparin vs acute event)
• Most coagulation based test for APLA
 Hexagonal phospholipid not affected
Warfarin
• Protein C and protein S
• Need to wait 3 weeks before testing protein S
• Most coagulation based APLA tests
Hypercoagulable states
Inherited
-“ The big five”
• Factor V Leiden
• Prothrombin gene mutation
• Protein S deficiency
• Protein C deficiency
• Antithrombin III deficiency
Factor V Leiden





Most common inherited risk factor
Often associated with other risk
factors e.g: homocysteine
Dramatic increase in risk with
estrogens ( OC Pills)
Hypercoagulability states
Acquired
 Cancer
 Antiphospholipid antibody syndrome
 Nephrotic syndrome
 Inflammatory bowel disease. Etiology:
inflammatory cytokines, low protein s
 Homocysteine
Clues to Acquired state : older age at the
time of onset, refractory to Warfarin , both
venous and arterial thromboembolism.
DVT and Cancer




Clues –
Bilateral DVTs, Arterial and venous
thrombosis and Warfarin refractory
thrombosis
DVT Diagnosis
Differential Diagnosis
• Cellulitis
• Knee pathology i.e. ruptured
synovial cyst
• Calf muscle strain
• Calf muscle hematoma
INVESTIGATIONS






Venogram : Gold standard. Rarely performed due
to accuracy of non invasive testing. Risks:
phlebitis, hypersensitivity, nephrotoxicity,
cardiotoxicity
Venous duplex ultrasound : non invasive,
accurate, first line study in moderate to high risk
DVT. Specificity : 95%. Sensitivity 97% for
proximal DVT and only 73% for distal DVT. Can
identify other pathology e.g.: calf haematoma,
bakers cyst, abscesses
MRI : 90% sensitive and specific for acute
Proximal DVT.
D-Dimer assays






D-Dimer should not be used as a screening test.
Can be used as a first line test in pts with low
pretest probability for DVT.
The utility of D-Dimer is mainly to reduce the
number of ultrasound testings.
The negative predictive value of a d-dimer assay
falls as the pretest probability for DVT increases.
An assay with a sensitivity of 80% has a negative
predictive value (NPV) of 97.6% in a low-risk
patient. However, the NPV of the same assay is
only 33% in high-risk patients with a pretest
probability of 90% for DVT.
CLINICAL ( PRETEST) PROBABILITY OF
DVT

WELLS DVT SCORE SYSTEM ( most popular )


















Clinical Parameter
Score

Active cancer (treatment ongoing, or within
6 months or palliative)
Paralysis or recent plaster immobilization
of the lower extremities
Recently bedridden for >3 d or major
surgery <4 wk
Localized tenderness along the distribution
of the deep venous system
Entire leg swelling
Calf swelling >3 cm compared to the asympto
- matic leg
Pitting edema (greater in the symptomatic leg)
Previous DVT documented
Collateral superficial veins (non varicose)
Alternative diagnosis (as likely or > that of DVT )

+1
+1
+1
+1
+1
+1
+1
+1
+1
- 2
Wells DVT Clinical Score ( contd )




Score of Zero – low probability ( 0 to
13%)
Score 1 – 2
probability)



- moderate probability
( 13 to 30%

Score > or = 3 - high probability
( 49 to
81%probability)
Complications - DVT






Pulmonary Embolism
Post obstructive syndrome : pain and
edema in the affected limb without
new clot formation
Chronic venous insufficiency
Rarely, thrombosis is massive,
causing vascular compromise of the
leg due to high venous pressure
(i.e., phlegmasia cerulea dolens).
DEEP VEIN
THROMBOSIS
TREATMENT
Treatment




Anticoagulation – heparin, warfarin,
lmwh – overlapping treatments
IVC filters – greenfield filter
OPTIMIZING WARFARIN THERAPY





Dosage to be individualized according to patient’s
INR response. Target INR -2-3
Use of large loading dose may lead to
hemorrhage and other complications.
Initial dose: 2-5 mg once daily
Maintenance dose: 2-10 mg once daily
Start heparin on day 1 and warfarin in the
evening of Day1 or on Day2. Heparin is usually
discontinued after 4-5 days. Before discontinuing,
ensure INR is in therapeutic range for 2 consecutive
days



Monitor daily until INR is in therapeutic range,
then 3 times weekly for 1-2 weeks, then less
often (every 4 to 6 weeks)
CONTARINDICATIONS AND
PRECAUTIONS - WARFARIN






Hypersensitivity to warfarin
Condition with risk of hemorrhage
Hemorrhagic tendency
Protein C & S deficiency
Vitamin K deficiency
SIDE EFFECTS







Hemorrhage
Skin necrosis
Microembolization
Teratogenecity : contraindicated
in pregnancy
Agranulocytosis, leukopenia,
diarrhea,
nausea, anorexia.
Uncomplicated DVT - OP management
Any reason for Hospitalization ?
potential probm with home Rx

No probm with Home Rx

Hospitalize
Give BID S.C LMWH + Warfarin
Ongoing Evaluation

Give 1st dose S.C LMWH Stat
Begin Warfarin in Doses 5mg QD
Assign a visiting nurse for twice
daily injs ( until the pt or family
member can inject ) Daily PT and
CBC with platelet count.
After atleast 5 days twice daily
injs of LMWH, a therapeutic INR
for 2 consecutive days – d/c LMWH
and follow warfarin carefully as per
routine for DVT.
Repeat physician evaluation by 5-7,
if no problem, repeat 1 wk later and
as per routine for DVT.
Reasons for hospitalization in DVT







Signs and symptoms of PE
Co-existing medical illness – anemia, lung
disease, previous bleeding, high risk of
bleeding or clotting
Depression or any other mental illness
that restricts pt’s co-operation and
compliance with the home Rx.
Insurance problem or other logistic
difficulty that limits pt’s access to home
Rx, nursing care, monitoring, food &
shelter
Duration of therapy

?
Duration depends on risk factor for
DVT









Risk of recurrence depends on type of risk factor.
If 1st DVT occurred after a major risk factor,
recurrence is 3% where as if it occurred after
minor risk factor recurrence is 10%  So, stratify
pts based on risk factor and then decide duration
Major transient risk factors : Major surgery,
major medical illness and leg casting.
Minor transient risk factors : OC Pills, HRT
High risk thrombophilias : Homzygos Prothrombin
gene mutation, Homozygos Factor v leiden,
antithrombin, protein c and protein s deficiencies
and APLA Syndrome
Low risk thrombophilias : Heterozygosity for
prothrobin gene mutation and Factor V leiden
Recommendations – Duration of Anticoagulant Rx in pts with DVT
Patient
Risk of recurrence
Duration of
characteristics
(%)
Therapy
- In the year after
discontinuation
Major transient risk
factor
b.Minor risk factor, no
thrombophilia
a.

Idiopathic event, no
or low risk
thrombophilia
d.Idiopathic event,
high risk
thrombophilia
e.More than one
idiopathic event
f.Cancer, other
ongoing risk factor
Ref: NEJM, 2004, 351

3%
<10% if risk factor
avoided. >10% if
persistent

c.

<10%
>10%
>10%
>10%

3 months
6 months
Until factor resolves
6 months
Indefinite
Indefinite
Indefinite. Consider
long term Rx with
LMWH in pts with
cancer
Contraindications - Anticoagulant
RX
Absolute contraindications:
 Active bleeding
 Severe bleeding diatheses or platelet count< 20,000/mm3
 Neurosurgery, ocular surgery or intracranial bleeding in
past 10 days
Relative contraindications:
 Mild/moderate bleeding diatheses or thrombocytopenia
 Recent major trauma
 Major abdominal surgery in past 2 days
 Brain metastases
 Gastro/genitourinary bleeding in past 2wks
 Endocarditis
 Severe hypertension at presentation ( SBP>200 and
DBP>120)
Case Study









A 79-year-old man is admitted to the medical ward 3 days status
post subdural hematoma drainage, C3 cervical spine fracture, and
fixation of multiple extremity fractures sustained in a motor
vehicle accident. The patient is now awake and oriented to person,
place, and time, but is a lower cervical spine incomplete
quadriplegic. Physical examination reveals some minimal sensation
in the legs, but no ability to move the extremities. There is a Foley
catheter in place that is draining yellow colored urine. Doppler
ultrasonography demonstrates a thrombus in the left popliteal
vein. The most important next step in the management of this
patient is
A. daily Doppler ultrasonography of the lower extremities
B. inferior vena cava filter placement
C. subcutaneous heparin
D. tissue plasminogen activator thrombolysis
E. warfarin
F. weekly ventilation/perfusion scans for a pulmonary embolus
Case Study










A 49-year-old man comes to clinic for follow up and monitoring of
his oral anticoagulation levels. The patient is postoperative day 62
from a left total knee replacement. On postoperative day number 2
he suffered a pulmonary embolism. He was placed on intravenous
unfractionated heparin and then oral warfarin. He was discharged
home with follow-up instructions to return to the clinic for
monitoring of his prothrombin time/international normalized ratio
(INR) every 3 weeks. On return to the clinic today his PT/INR is
found to be 22.4/7.3. His physical examination is unremarkable.
The most appropriate management at this time is to
A. admit the patient to the hospital
B. instruct the patient to discontinue warfarin and return in 1
week
C. instruct the patient to discontinue warfarin week until his next
visit in 3 weeks
D. give protamine sulfate, intravenously
E. give vitamin K and follow up with the patient at his next visit
Thrombosis in
Pregnancy
Preventing Thrombosis in
Pregnancy
Give prophylaxis with heparin during pregnancy and
continued heparin or warfarin for 6 weeks
postpartum in women with high risk of recurrent
thrombosis:
 Recent thromboembolism (within the previous 6
months)
 An indication for lifelong anticoagulation
 Previous thrombosis with a thrombophilia
associated with a high risk of recurrent
thrombosis (homozygotes for factor V Leiden,
homozygotes for the prothrombin gene mutation,
or antiphospholipid antibodies
 A history of recurrent thromboembolism




Consider heparin prophylaxis antepartum
and intrapartum, and heparin or warfarin
prophylaxis postpartum in women with
one previous thromboembolic event even
without underlying thrombophilia, family
history, or other risk factors (such as
obesity, smoking, recent surgery,
immobilization, advanced age, and parity).
Advise pregnant women, especially those
with a history of thrombosis or
thrombophilia, to stop smoking
Rxng Thrombo-embolism in
Pregnancy




Avoid the use of warfarin during
pregnancy because it crosses the placenta
and is associated with teratogenesis and
increased fetal morbidity and mortality.
Treat with LMWH @ therapeutic doses. 
Continue long-term prophylaxis against
recurrent venous thromboembolism after
the postpartum period and completion of
therapy using warfarin, aiming for a target
INR of 2 to 3 ( usually 6 mos if no other
hx of thrombophilia is present)
Anemias
Anemias



















Look At MCV  Microcytic, Macrocytic & Normocytic
Iron deficiency – Colon ca scenarios, Fibroids, malabsorption
syndromes (Celiac sprue), Gastric bypass surgery (Iron is
mainly absorbed in the duodenum and jejunum)
B12, Folic acid deficiency
B12 deficiency in the elderly
Sideroblastic Anemia
Thalassemias
Autoimmune Hemolytic Anemias – initial rx sterods, then
IVIG, if unresponsive splenectomy
Hereditary Spherocytosis
G6PD Deficiency – sulfa, dapsone, primaquine
Sickle Cell Anemia – Crisis, Aplastic crisis, Acute chest
syndrome, Avascular necrosis, osteomyelitis,
autospenectomy, Folate supplement importance
Aplastic Anemia
Anemia of Renal Disease – Give EPO if Hgb < 10 and
supplement iron if Tsat < 20%
Anemia of Chronic Disease
Bone marrow biopsy – indications in Anemia
Microcytic Anemias






Importance of RDW
Peripheral smear – increase in
central pallor
Iron studies – IRON/TIBC, Ferritin
Differentiate from AOCD – in AOCD,
ferritin normal/ high, iron low, tibc
low
Rx – Iron Deficiency









Correct underlying cause – Menorrhagia,
Malabsorption
Recommend Hysterectomy for fibroids
with menorrhagia in a patient who has
completed her family.
Oral Iron Supplements – preferred
Side effects oral iron – nausea,
constipation
If too many side effects and severe Fe
deficiency – consider Iron dextran IV
Hereditary Spherocytosis





Lack of central pallor
Osmotic Fragility test
Rx : blood transfusions, splenectomy
Watch secondary hemochromatosis –
use Desferrioxamine

















A 32-year-old man of Italian descent is evaluated for a routine preemployment physical examination. He has always been healthy, and his
physical examination is normal.
Laboratory Studies
Hematocrit 35%
Mean corpuscular volume 63 fL
Leukocyte count 6800/μL
Reticulocyte count 40,000/μL (0.7% of erythrocytes)
Platelet count 270,000/μL
Results of fecal occult blood testing are negative. Peripheral blood smear
shows microcytosis and many target cells.
Which of the following is the best diagnostic test to evaluate the
cause of the anemia and microcytosis?
( A ) Coombs' direct antiglobulin test
( B ) Measurement of hemoglobin A2 level
( C ) Glucose-6-phosphate dehydrogenase screen
( D ) Measurement of serum iron, total iron-binding capacity, and ferritin
levels












A 22-year-old female college student of Greek descent is evaluated
because she tires easily and has had palpitations
when playing tennis for the past 3 months. She reports no night sweats or
weight loss, and her family history is negative for anemia. She has two
healthy siblings.Physical examination is unremarkable. A complete blood
count shows a hemoglobin of 9.0 g/dL with a hematocrit of 28% and an
erythrocyte count of 3.7 million/μL. Erythrocyte distribution width is
elevated at 17% (normal range 10.5% to 14.5%). The neutrophil,
lymphocyte, and platelet counts are normal.
Which of the following is the most appropriate initial step in the
management of this patient?
( A ) Perform hemoglobin electrophoresis for hemoglobin A2 and F.
( B ) Perform complete blood counts for siblings.
( C ) Measure serum ferritin.
( D ) Perform ultrasonography of the spleen.
( E ) Perform Southern blot analysis
Key Points
An elevated erythrocyte distribution width is
consistent with iron-deficiency anemia.


Measurement of serum ferritin can differentiate
iron-deficiency anemia from the anemia of
thalassemia trait.

Macrocytic Anemias








Vitamin B12 def
Folic acid def
Alcohol/ Zidovudine
Hypothyroidism
Subtle clues for Vit b12 def  ataxia,
neurological symps, psychosis, dementia
Pernicious anemia – screen all B12 def pts
with anti-parietal cell abs. ( Rx is IM b12
for life )
Hemolytic Anemias










Labs that suggest Hemolysis  Indirect
bilirubin, LDH, Reticulocyte count,
Haptoglobin
Peripheral smear  help to r/o
microangiopathic hemolysis
(Schistocytosis), increased reticulocytes
Urine Hemosiderin  elevated only in
uintravascular hemolysis
Direct coomb +ve  Autoimmune
Hemolysis.
Concomitant thrombocytopenia may
suggest TTP, DIC, EVANS syndrome
Microangiopathic Hemolysis






Characterized by schistocytosis –
fragmented RBCs on peripheral
smear
Seen in TTP, DIC, HUS, HELLP
syndrome, CAPS, malignant
hypertension
Macroangiopathic hemolysis – also
can have schistocytes eg: prosthetic
valves.
Autoimmune Hemolytic Anemias








Direct coombs +ve
Microspherocytes on peripheral smear
Urine hemosiderin –ve
LDH elevated, Hapto may be low
Retic count increased
Rx : Steroids, IVIG, Splenectomy
Recognize Autoimmune hemolysis in CLL















A 63-year-old man with stage I chronic lymphocytic leukemia (CLL) is evaluated for
increasing dyspnea on exertion that has developed over the past 2 weeks. He
currently takes no medications. On physical examination, he has pale conjunctivae
and scattered axillary and inguinal lymphadenopathy that are unchanged from his
last examination 1 year ago. His is afebrile. Pulmonary examination is normal except
for tachypnea.
Laboratory Studies
Hematocrit 18%
Leukocyte count 12,000/μL (25% polymorphonuclear leukocytes, 75%lymphocytes)
Platelet count 285,000/μL
A peripheral blood smear shows spherocytes, a reticulocyte count of 10%,
polychromatophilia, smudge cells, and normal-appearing lymphocytes with no
schistocytes. Hematocrit 1 year ago was 46%.
Which of the following is the most likely cause of this new onset of anemia?
( A ) Conversion of CLL to acute lymphoblastic leukemia
( B ) Autoimmune hemolytic anemia
( C ) Disseminated intravascular coagulation
( D ) Marrow infiltration by CLL
( E ) Conversion to a large-cell lymphoma (Richter's syndrome)











A 27-year-old black man is admitted to the hospital for treatment
of community-acquired pneumonia, for which he received
erythromycin. On the second hospital day, he is lethargic and has
easy fatigability, a temperature of 38.4 °C (101.1°F), and scleral
icterus. The patient had a similar episode during a childhood ear
infection. His two brothers have had similar problems, though his
sister has not. His hematocrit, which was 40% on admission, is
now 32%. His reticulocyte count is 140,000/μL (4% of
erythrocytes).Liver studies show a serum total bilirubin
concentration of 5.4 mg/dL and a direct bilirubin concentration of
1.4 mg/dL.
A peripheral blood smear is shown.
Which of the following is the most appropriate diagnostic
study at this time?
( A ) Sickling test
( B ) Glucose-6-phosphate dehydrogenase measurement
( C ) Hemoglobin electrophoresis
( D ) Erythrocyte pyruvate kinase activity measurement
( E ) No further testing
Key Point
Following an acute hemolytic event due to
glucose-6-phosphate dehydrogenase deficiency
(G6PD),G6PD levels will be normal. Testing
should be delayed for 3 to
4 weeks.
Anemias













A 19-year-old woman comes to the student health service
complaining that since the new semester has begun, she finds
herself unable to focus and concentrate as well as before. She
attributes this largely to feeling fatigued. She denies any other
symptoms such as sadness, sleeplessness, or loss of libido. She
has no other medical issues. Her medications are only oral
contraceptive pills. She has never been pregnant and denies
current pregnancy. She has a history of long menses, often lasting
8 days. Laboratory studies show:
Hct 31%
MCV 69um3
Ferritin 10mcg/L
The most appropriate next step is to
A. administer iron, intravenously
B. administer vitamin B12, intravenously
C. advise her to take folate tablets
D. change her oral contraceptive formulation to estrogen only
E. tell her to take oral iron tablets











Anemias

A 5-year-old boy is brought to the clinic for a periodic health maintenance
examination. He is generally healthy, enjoys school, plays well with his
siblings and with other children his age. He and his family live in a housing
development down the street that was built 10 years ago. Since his mother
usually works until late in the evening, he tends to spend a lot of time at a
friend's apartment in an old, dilapidated housing development nearby. You
notice that he has unusually pale skin and mucus membranes and so you
inquire about related symptoms. The mother tells you that she has noticed
that he is significantly more tired than his siblings and he has been a "bit
irritable" lately but she "didn't think nothing of it." He is up-to-date on all of
his immunizations. There is no family history of blood disorders, however
several of his playmates "are anemic." You decide to order hemoglobin,
hematocrit, and a peripheral blood smear and schedule a follow-up visit in 1
week. He returns for his next appointment and you review the results of the
laboratory studies. His hemoglobin is 9.5 g/dL, hematocrit is 30%, and the
peripheral blood smear shows microcytic red blood cells with basophilic
stippling. The most appropriate next step is to
A. administer ferrous sulfate, orally
B. administer dimercaprol, orally
C. administer edetate disodium, orally
D. determine B12 levels
E. determine blood lead levels
F. obtain an abdominal radiograph
G. order hemoglobin electrophoresis
H. Check Erythrocyte Zinc Protoporphrin levels

















During a routine screening for mild dementia, a 78-year-old male resident of a
nursing home is found to have a low serum vitamin B12 level. He does not have
fatigue, dyspnea, chest pain, dizziness, unsteadiness, or paresthesias. His
medical history is significant for coronary artery disease, for which he underwent
coronary stent placement 5 years ago. His medications include metoprolol and
atorvastatin. Physical examination does not indicate any symptoms of vitamin B12
deficiency, such as glossitis or impaired sensation.
Laboratory Studies
Hematocrit 43%
Mean corpuscular volume 94 fL
Serum creatinine 1.4 mg/dL
Serum vitamin B12 level 200 pg/mL
Which of the following is the most appropriate next step in the management
of this patient?
( A ) Weekly vitamin B12 injections
( B ) Intrinsic factor antibody test
( C ) Serum methylmalonic acid level
( D ) Schilling test
Subclinical B12
Deficiency
- May not have any symps
- Elevation of MMA is a sensitive
marker of significant deficiency
- Treat it!
Sickle Cell Anemia














Sickle cell trait/ Sickle cell anemia
Sickling test : sickle cell screen
C/F:Sickle cell crises
Acute Chest syndrome
Infections
Dactylitis
Aplastic Crises : folic acid deficiency vs. Parvo virus
Avascular Necrosis hip  get MRI
Labs : HGB electrophoresis, Peripheral smear – howell-jolley
bodies, stress lymphocytosis, increased retic count, anemia,
hemolysis, elevated indirect bilirubin
Management :
Acute pain crises  manage with IV Fluids, analgesics, septic
w/u, electrolyte repletion, watch for substance abusers ( clues
are labs)
Recurrent Acute pain crises  more than 3-4 episodes per
year, use Hydroxyurea. D/W pt benefits vs. toxicity of hydrea
Acute Chest Syndrome  O2, analgesics, Exchange
Transfusion








A 17-year-old male patient with homozygous sickle cell
anemia was admitted to the hospital for hydration and
analgesia of a painful crisis. On hospital day 3, his condition
deteriorated, with the onset of fever, new chest pain, and
dyspnea. On physical examination, he has a temperature of
38.9 °C (102 °F), pulse rate of 120/min, and respiration
rate of 32/min. He is using accessory muscles of inspiration
and has crackles bilaterally. Chest radiographs show new,
bilateral pulmonary infiltrates and a normal-sized heart.
Pulse oximetry shows an oxygen saturation rate of 80% on
room air. Which of the following is the most important
immediate therapeutic option?
( A ) Amoxicillin
( B ) Hydroxyurea
( C ) Diuretics and digoxin
( D ) Partial-exchange transfusion
( E ) Polyvalent pneumococcal vaccine
Key point
The acute chest syndrome is characterized by
chest pain, fever, diffuse pulmonary infiltrates,
and hypoxia in a patient with sickle cell anemia.


The most effective therapy for the acute chest
syndrome in sickle cell anemia is partialexchange transfusion to lower the hemoglobin
S.

Splenectomy




Prior to splenectomy Patients should
be immunized with pneumococcal,
HIB and meningococcal vaccines
Daily oral penicillin prophylaxis for
splenectomized patients.

















A 56-year-old man is evaluated for fatigue during a routine office visit. His history is
significant for diverticulosis,hypertension, and supraventricular tachycardia, for
which he takes aspirin, metoprolol, and ramipril. On physical examination, he is
afebrile, his blood pressure is 120/80 mm Hg, and pulse rate is 80/min. No
abdominal tenderness,splenomegaly, or lymphadenopathy is noted.
Laboratory Studies
Hemoglobin 8 g/dL
Mean corpuscular volume 76 fL
Leukocyte count 11,200/μL
Platelet count 847,000/μL
Which of the following is the most appropriate next diagnostic test for this
patient?
( A ) Fluorescence in situ hybridization analysis of blood for Philadelphia chromosome
( B ) Examination of bone marrow aspirate
( C ) Serum ferritin measurement and fecal occult blood testing
( D ) Repeated complete blood count in 2 weeks after discontinuation of all current
medications
( E ) Bleeding-time measuremen
Ans. Reactive Thrombocytosis








Iron deficiency
Malignancy
Major Surgery
Acute blood loss
Primary thrombocytosis can be caused by clonal disorders, such
as chronic myelogenous leukemia (CML), polycythemia vera, and
essential thrombocythemia. CML is characterized by the presence
of the Philadelphiachromosome in blood and marrow myeloid
cells. Patients with CML typically have a high leukocyte count,
high or normal platelet and erythrocyte counts, and
splenomegaly. The absence of leukocytosis and splenomegaly and
thepresence of severe microcytic anemia make CML unlikely
Essential thrombocythemia may be associated with thrombotic
and hemorrhagic complications. Patients with essential
thrombocythemia have a platelet count of greater than
600,000/μL without known causes for reactive thrombocytosis;
therefore, iron deficiency precludes this diagnosis.
Bleeding Disorders








A 48-year-old female nurse is evaluated for hematuria. One week
ago, she experienced dysuria, for which cephalothin was
prescribed. Her history is remarkable only for an uneventful
tonsillectomy as a child and removal of a benign ovarian cyst at
age 36; there is no family history of bleeding. The patient does not
smoke or use alcohol or recreational drugs. She has a normal diet,
and her weight has been stable. She has a history of depression
that has been treated with numerous antidepressants, but she is
not taking any medications currently. On physical examination, she
has two ecchymoses on her lower extremities. Laboratory
evaluation includes a prothrombin time (PT) of 28.2 sec (INR 4.2),
activated partial thromboplastin time (PTT) of 45 sec (normal < 35
sec), normal thrombin time, negative D-dimer, and normal
complete blood count and platelet count. Liver enzymes are within
normal limits, and serum albumin is 4.1 g/dL. An inhibitor-screen
mixing study shows complete correction of the PT and PTT after
mixing of patient and control plasma at a 1:1 ratio. The factor VII
level is 6%, factor IX level is 10%, factor VIII level is 110%, and
factor V level is 95% (reference range 60% to 150%).
Which of the following is the most likely diagnosis?
( A ) Acquired factor VIII inhibitor
( B ) Lupus anticoagulant
( C ) Hemophilia B (Christmas disease)
( D ) Surreptitious warfarin ingestion
Key Point
In severe vitamin K deficiency, the prothrombin
time and the Partial thromboplastin time are
prolonged.
In mild vitamin K deficiency (including
therapeutic warfarin), only the prothrombin
time is prolonged

Von Willebrands
Disease
Screen for VWd








Screen for VWD in women with menorrhagia and no
gynecologic reason for heavy bleeding, in patients with
unexplained procedural bleeding, and in individuals with a
personal or family history or physical evidence of
mucocutaneous bleeding.
Remember VWd can be asymptomatic for a long time with
no hx at all and may just present with excess bleeding
during surgical procedures
Conduct a first-line coagulation screening work-up,
including PT, PTT, CBC/platelets, and thrombin time.  If
PTT is prolonged and other initial test results are normal,
suspect VWD as a possible diagnosis ( BT is prolonged too.
BT is N in Hemophilia ) ( Remember d/d for isolated PTT
prolongation is also thrombotic condition APLA/ LA – This
manifests by thrombosis though, unlike VWd which is
bleeding)
Initial tests for the diagnosis or exclusion of VWD include
VWF function, VWF:Ag, and factor VIII activity and should
be done more than once.
Rx - VWd




In treatment of significant bleeding
events or for prevention of bleeding
during invasive procedures or
surgeries  use Desmopressin
Remember Tachyphylaxis with
desmopressin ( in case of continued
bleeding after desmopressin , use
Cryoprecipitate/ VWF concentrate)
Uremic Bleeding








Bleeding time is the best predictor
for tendency to bleed.
Desmopressin – Rx of choice in
preventing bleeding for surgical
procedures or in Rx of Bleeding
Conjugated Estrogens
Blood transfusion to keep hct high
Recombinant Factor VIIa
Hemophilia
Screen for hemophilia A and B in individuals with:
 A history of bleeding
 Family members known to have hemophilia or be
hemophilia carriers
 Unexplained aPTT prolongation ( BT normal)
 DX : If APTT prolonged, do mixing study. Correction
suggests Factor deficiency.
 Then obtain quantitative factor VIII/ IX levels and activity.
 In patients with prolonged PTT and decreased factor VIII
activity who have no family hx of hemophilia a, obtain VWF
level to r/o VWd
Hemophilias can present with deep bleeding ( Hemarthrosis)
apart from superficial bleeding ( mucosal bleeding,
ecchymoses, bruises with minimal trauma)
RX: factor VIII/ IX concentrates ( A – Factor VIII def, B –
FACTOR ix DEF)
Thrombocytopenias
Thrombocytopenias











Drug Induced : HIT, GP IIb IIIA inhibitors
( abciximab, eptifibitide), Linezolid,
Clopidogrel
Always r/o Pseudothrombocytopenia – this
is our first step.
Congenital thrombocytopenia ( Some are
microthrombocytopenias)
Consumption Thrombocytopenias : TTP,
DIC, Massive Bleeding, Sepsis
ITP
ITP + AIHA ( Coombs +ve, Evans
syndrome)
Platelet transfusions - Indications







Platelets < 15k
Platelets<50k + ICH/Life threatening
hemorrhage
Platelets > 50k  no platelets ( all we
need is 50k platelets for enough clotting.
So, no need of transfusions for
platelets>50k)
Avoid platelets in TTP/ DIC unless severe
bleeding which is unlikely manifestatuion
in these cases.
Idiopathic thrombocytopenic purpura

















Previous CBC is a clue
Rule out other causes – drugs, liver disease, HIV, HEP-C, APLA, SLE
etc
HIV related ITP responds to HAART/ Zidovudine most studied.
Rx – mild ITP is often associated with a good prognosis.
Platelet counts >50 × 109 cells/L with no history of bleeding 
careful observation with regular visits and platelet count
determinations
Prednisone f Plts < 30k  If there is a response with a
normalization of the platelet count over 1 to 2 weeks, taper by 5 to
10 mg/week.
IVIG if immediate response is required i.e; before surgical
procedures, or in initial treatment for patients with severe bleeding
AND also, in patients who are refractory to treatment with
glucocorticoids. Remember, response short-lived.
Consider RhIG as outpatient therapy: In patients who are Rh(+)
and have not had a splenectomy , For patients in whom
splenectomy is not feasible and along with IVIG in patients in
whom steroids have failed
Splenectomy if no response to steroids (persistent platelet counts
<10 × 109 cells/L for 4 to 6 weeks and for patients who have had
ITP for 3 months with persistent severe thrombocytopenia
(platelet count <30 × 109 cells/L) despite treatment) or Relapse
after tapering Steroids

Can try immunosuppressants (AZA) or Rituximab if
splenectomy and Steroids are unsuccessful
HIT















Usually occurs with in 5-7 days after exposure to heparin.
Immediate HIT is possible if there was prior exposure in
past 3 months enabling circulating antibodies
Heparin induced Anti-platelet Abs  screening test
Serotonin Release Assay  confirmatory test
Base your decision most on clinical probability rather than
these tests.
Features  deep vein thrombosis, PE, Arterial thrombosis
in a setting of acute thrombocytopenia after exposure to
heparin.
Rx  Once HIT is suspected/ diagnosed , must be treated
Rx  Argatroban, leperidin overlap with warfarin
Lepiridin difficult to monitor
Argatroban adds to INR. So, safely discontinued after
warfarin started and INR of 4.0 is reached. Repeat INR in 6
hrs after discontinuing Argatroban to confirm therapeutic
range INR ( 2 to 3)
Case Study








A 67 y/o man in previously good health is
hospitalized with 2 day hx of fever and decreased
consciousness. Temp is 103 F, HR 140, BP 88/50.
There is no bleeding. The hgb 12.1 gm%, wbc
29,000, platelets 20,000/ul. Which of the
following should be obtained next?
A) Bone marrow biopsy
B) Factor VIII level
C) Measurement of platelet associated IgG
D) Measurement of D-Dimer and fibrinogen
E) Bleeding time
Anemias


A 21-year-old man with no significant past medical history
presents to office with complaints of blood in his urine and
mucosal bleeding while brushing his teeth. He denies any drug or
alcohol use. He has no family history of bleeding disorders.
Petechiae are noted in the oral cavity, as is dried blood in the
nostrils. Laboratory studies show the following: Hematocrit 32%;
white blood cell count 8,000/mm3 with 60% neutrophils; platelet
count 13,000; PT 13 seconds; PTT 28 seconds; LDH 1,200 U/L;
elevated indirect bilirubin. Coombs' test is positive; abdominal
examination is normal; and the peripheral smear shows
spherocytes.
What is the most likely diagnosis?
(A) Gordon's syndrome
(B) Bernard-Soulier syndrome
(C) Felty's syndrome
(D) Thrombotic thrombocytopenic purpura
(E) Evans' syndrome
(F) Idiopathic thrombocytopenic purpura (ITP)
Myeloproliferative Disorders






Polycythemia vera ( differentiate
from secondary polycythemia)
Essential thrombocytosis
CML
Myelofibrosis ( “tear drop” cells)
Polycythemia Vera











Measure RBC mass to r/o relative polycythemia
Differentiate from Secondary polycythemia – R/o
hypoxia which is the most common cause.
Presence of leucocytosis and thrombocytosis as
well as spelenomegaly are big clues that this
could be primary.
EPO level
JAK-2 mutation
Pruritis during shower often seen in primary
polycythemia due to histamine release from mast
cells
Bone marrow biopsy if EPO is low or normal in
presence of polycythemia
Secondary Polycythemia








R/o Relative Polycythemia secondary to
hemoconcentration/ dehydration  repeat
CBC after Hydration
Chronic Hypoxia : COPD, high altitudes,
OSA, Obesity hypoventilation, intracardiac shunts, smoking  EPO elevated/
normal
Neoplasms : RCC, Pheochromocytoma,
Cushings disease, cerebellar
hemangioblastoma, hepatoma
Endocrine : Cushings disease













C/F and Rx

Pruritis in Polycythemia vera
Ruddish complexion, erythematous rashes on
abdomen, extremities, purpura.
Pts have increased risk of both thrombosis and
bleeding
Phlebotomy to keep hct < 45%
Patients with symptomatic polycythemia should
undergo immediate Phlebotomy
Headache, Dizziness, chestpain
Blurred vision, papilledema, hypertensive
emergencies are indication for immediate
Phlebotomy.
Begin cytoreductive therapy concomitantly with
phlebotomy in cases of P.Vera (Hydroxyurea).
Hematological Malignancies








Leukemias – ALL, AML, CML, CLL
Know when to treat CLL ( Anemia,
thrombocytopenia, Rx – Fludaribine,
Chlorambucil)
CML – Studies, Rx
AML  Know M3, associations and Rx
Lymphomas  Hodgkins, NHL
Secondary malignancies after radiotherapy
( Hodgkins survivors )  follow up
mammograms
Peripheral smear
Ans


Plasma cell/ eccentric nucleus
Multiple Myeloma
D/D
MGUS
Solitary Plasmocytoma
POEMS syndrome (peripheral neuropathy, organomegaly,
endocrinopathy, monoclonal gammopathy, skin changes, as well as
other paraneoplastic features and osteosclerotic lesions)
Waldenström macroglobulinemia
MGUS









M-Protein < 3gm%
Bone Marrow plasma cells < 10%
Normal hgb, creatinine and serum calcium
Normal bone survey ( x-rays not bone
scan)
RX : may progress to MM. There’s no
evidence that chemotherapy reduces
progression – so don’t treat
Observation
Solitary plasmacytoma









Single lytic lesion in the bone
No evidence of anemia, renal
insufficiency or high calcium
M-protein may or may not be
elevated
75% progression to MM in 10 yrs
No chemotherapy is recommended
Rx is excision and Radiation
Smoldering Myeloma


Characteristics
• Serum M protein >3 g/dL and/or bone marrow
plasma cells ≥10%
• Absence of anemia, renal failure,
hypercalcemia, lytic bone lesions



Disease Management
• Observation with treatment beginning at
disease progression
• Bisphosphonates
• Supportive care
• Participation in clinical trial
Symptomatic MM




Patients with symptomatic myeloma
require immediate treatment.
Characteristics
•
•
•



Presence of serum/urine M protein > 3gm%
Bone marrow plasmacytosis (usually >30%)
SYMPTOMS+  Anemia, renal failure,
hypercalcemia, or lytic bone lesions

Treatment — Thalidomide + dexa,
lenalidomide, Melphalan
Waldenstroms Macroglobulinemia
Incidence and clinical features





Increased IgM. IgG may be decreased
1,500 cases/year in USA
Median age -, 63 yrs
Presenting symptoms
•
•
•
•
•
•

Weakness and fatigue
Hemorrhagic manifestations
Weight loss
Neurologic symptoms
Visual disturbances
Raynauds phenomenon

44%
44%
23%
11%
8%
3%
Waldenstroms Macroglobulinemia:
Clinical Features


Tumor infiltration

• Bone marrow
• Splenomegaly
• Lymphadenopathy



Circulating IgM
•
•
•
•



Hyperviscosity syndrome
Cryoglobulinemia
Cold agglutinin disease
Bleeding disorders

Tissue IgM

• Neuropathy

90%
38%
30%
Waldenstorms macroglobulinemia






Symptoms of hyperviscocity syndrome ( Headache,
chestpain, sob, blurred vision, papilledema)
Rx:
Plasmapheresis for circulating IgM complications
Alkylating-agent based therapy (50-70% response rate)
•
•
•
•



Chlorambucil and prednisone
Cyclophosphamide
Melphalan
CHOP (Cyclophosphomide, adriamycin, vincristine, prednisone)

Nucleoside analogues (80-90% response rate)
- Fludarabine
- 2-Chloro-deoxyadenosine (2-CdA)
Hematological Emergencies






TTP
LEUCOSTASIS ( CML, AML)
Acute Tumor Lysis Syndrome
HIT
Hyperviscocity Syndrome
Transfusion Medicine
Transfusion Reactions










Febrile non hemolytic reactions
Acute hemolytic transfusion reactions
Delayed hemolytic transfusion
reactions
Anaphylactic reactions
Urticarial reactions
TRALI
Post transfusion purpura
Graft versus host disease
Case Study








A 43 y/o m with severe acquired aplastic anemia
has not responded to immunosuppressive agents.
He has a HLA identical brother who has been
cleared as a donor for his planned allogeneic
stem cell transplant. They are both CMV negative.
Which of the following will be prevented by using
irradiated cellular blood products to this patient?
A) Graft versus Host disease, Transfusion related
B) CMV disease
C) Alloimmunization
D) Hemolytic transfusion reaction
E) Febrile non hemolytic transfusion reaction










A 57-year-old woman (gravida 5, para 5) was admitted to the
hospital with a bleeding duodenal ulcer for which she was given
three units of packed erythrocytes. A regimen of antacids and
cimetidine was initiated, and the patient was discharged after 3
days. Two days later, she returns to the hospital because of
weakness, increased malaise, and dark urine. Her hematocrit,
which was 37% at discharge, is now 26%. Liver studies show a
total serum bilirubin of 6.1 mg/dL, with a direct value of 2.3
mg/dL, and a lactate dehydrogenase concentration of 467 U/L.
Renal function is normal. Stools and a gastric lavage are negative
for blood. The peripheral blood smear shows numerous
microspherocytes.
Which of the following is the most appropriate next step in
the management of this patient?
( A ) Corticosteroid therapy
( B ) Blood typing re-evaluation and cross-match
( C ) Osmotic fragility test
( D ) High-dose immunoglobulin infusion
( E ) Emergency surgical consultation
Key points
1.

A delayed hemolytic transfusion reaction occurs
several dayS after transfusion and is characterized
by an elevated serum lactate dehydrogenase level
and a high indirect bilirubin level along with
decreasing hematocrit and the presence of
microspherocytes.

2. A repeated cross-match is likely to detect
antibodies missed on the original cross-match.

More Related Content

What's hot

New oral anticoagulants (NOAC) WATAG guidelines
New oral anticoagulants (NOAC) WATAG guidelinesNew oral anticoagulants (NOAC) WATAG guidelines
New oral anticoagulants (NOAC) WATAG guidelinesSCGH ED CME
 
Case presentation ICH & Prosthetic MV
Case presentation ICH & Prosthetic MVCase presentation ICH & Prosthetic MV
Case presentation ICH & Prosthetic MVKamal Mergani
 
Oral anticoagulant
Oral anticoagulantOral anticoagulant
Oral anticoagulantMashiul Alam
 
Newer anticoagulants
Newer anticoagulantsNewer anticoagulants
Newer anticoagulantsDeep Chandh
 
Every thing regarding Warfarin.
Every thing regarding Warfarin. Every thing regarding Warfarin.
Every thing regarding Warfarin. basheerabdelrahman
 
New oral anticoagulant shivaomfinal noac
New oral anticoagulant shivaomfinal noacNew oral anticoagulant shivaomfinal noac
New oral anticoagulant shivaomfinal noacShivaom Chaurasia
 
Dabigatran
DabigatranDabigatran
Dabigatranhospital
 
Newer Oral Anticoagulants In Atrial Fibrillation - Dr Vivek Baliga
Newer Oral Anticoagulants In Atrial Fibrillation - Dr Vivek BaligaNewer Oral Anticoagulants In Atrial Fibrillation - Dr Vivek Baliga
Newer Oral Anticoagulants In Atrial Fibrillation - Dr Vivek BaligaDr Vivek Baliga
 
Newer oral anticoagulant 8.9.16
Newer oral anticoagulant 8.9.16Newer oral anticoagulant 8.9.16
Newer oral anticoagulant 8.9.16DR ANUP PETARE
 
Oral anticoagulants ppt
Oral anticoagulants ppt Oral anticoagulants ppt
Oral anticoagulants ppt Shalini Garg
 
Oral anti coagulants ppt
Oral anti coagulants pptOral anti coagulants ppt
Oral anti coagulants pptDr Renju Ravi
 
Anticoagulant, antithrombotic and anti platelet drugs
Anticoagulant, antithrombotic and anti platelet drugsAnticoagulant, antithrombotic and anti platelet drugs
Anticoagulant, antithrombotic and anti platelet drugsraj kumar
 
Should noacs replace warfarin
Should noacs replace warfarinShould noacs replace warfarin
Should noacs replace warfarinSameh Sadek
 
Mechanism of Acenocoumarol Interaction and Management
Mechanism of Acenocoumarol Interaction and Management Mechanism of Acenocoumarol Interaction and Management
Mechanism of Acenocoumarol Interaction and Management Anees Sukkur
 

What's hot (20)

New oral anticoagulants (NOAC) WATAG guidelines
New oral anticoagulants (NOAC) WATAG guidelinesNew oral anticoagulants (NOAC) WATAG guidelines
New oral anticoagulants (NOAC) WATAG guidelines
 
Anticoagulants
AnticoagulantsAnticoagulants
Anticoagulants
 
Case presentation ICH & Prosthetic MV
Case presentation ICH & Prosthetic MVCase presentation ICH & Prosthetic MV
Case presentation ICH & Prosthetic MV
 
Oral anticoagulant
Oral anticoagulantOral anticoagulant
Oral anticoagulant
 
Oral anticoagulant
Oral anticoagulant Oral anticoagulant
Oral anticoagulant
 
Newer anticoagulants
Newer anticoagulantsNewer anticoagulants
Newer anticoagulants
 
Every thing regarding Warfarin.
Every thing regarding Warfarin. Every thing regarding Warfarin.
Every thing regarding Warfarin.
 
New oral anticoagulant shivaomfinal noac
New oral anticoagulant shivaomfinal noacNew oral anticoagulant shivaomfinal noac
New oral anticoagulant shivaomfinal noac
 
Dabigatran
DabigatranDabigatran
Dabigatran
 
xaban anticoagulation
xaban anticoagulationxaban anticoagulation
xaban anticoagulation
 
Newer Oral Anticoagulants In Atrial Fibrillation - Dr Vivek Baliga
Newer Oral Anticoagulants In Atrial Fibrillation - Dr Vivek BaligaNewer Oral Anticoagulants In Atrial Fibrillation - Dr Vivek Baliga
Newer Oral Anticoagulants In Atrial Fibrillation - Dr Vivek Baliga
 
Newer oral anticoagulant 8.9.16
Newer oral anticoagulant 8.9.16Newer oral anticoagulant 8.9.16
Newer oral anticoagulant 8.9.16
 
Oral anticoagulants ppt
Oral anticoagulants ppt Oral anticoagulants ppt
Oral anticoagulants ppt
 
A Case of Warfarin induced SDH
A Case of Warfarin induced SDHA Case of Warfarin induced SDH
A Case of Warfarin induced SDH
 
Anticoagulants
 Anticoagulants Anticoagulants
Anticoagulants
 
Oral anti coagulants ppt
Oral anti coagulants pptOral anti coagulants ppt
Oral anti coagulants ppt
 
Anticoagulant, antithrombotic and anti platelet drugs
Anticoagulant, antithrombotic and anti platelet drugsAnticoagulant, antithrombotic and anti platelet drugs
Anticoagulant, antithrombotic and anti platelet drugs
 
Should noacs replace warfarin
Should noacs replace warfarinShould noacs replace warfarin
Should noacs replace warfarin
 
Anti coagulant(warfarin)
Anti coagulant(warfarin)Anti coagulant(warfarin)
Anti coagulant(warfarin)
 
Mechanism of Acenocoumarol Interaction and Management
Mechanism of Acenocoumarol Interaction and Management Mechanism of Acenocoumarol Interaction and Management
Mechanism of Acenocoumarol Interaction and Management
 

Viewers also liked

N.couse case study_hit
N.couse case study_hitN.couse case study_hit
N.couse case study_hitNiccole Couse
 
VTE & Duration of Anticoagulation
VTE & Duration of AnticoagulationVTE & Duration of Anticoagulation
VTE & Duration of AnticoagulationVein Global
 
Dvt prophylaxis
Dvt prophylaxisDvt prophylaxis
Dvt prophylaxisredaahmed
 
TARTING YOUR VENOUS ACCESS PROGRAM
TARTING YOUR VENOUS ACCESS PROGRAMTARTING YOUR VENOUS ACCESS PROGRAM
TARTING YOUR VENOUS ACCESS PROGRAMPAIRS WEB
 
Lymphoproliferative disorders
Lymphoproliferative disordersLymphoproliferative disorders
Lymphoproliferative disordersTai Alakawy
 
Ultrasound Assessment Of Chronic Venous Disease
Ultrasound Assessment Of Chronic Venous DiseaseUltrasound Assessment Of Chronic Venous Disease
Ultrasound Assessment Of Chronic Venous Diseasejavier.fabra
 
Modern management of dvt dr. sharfuddin chowdhury
Modern management of dvt dr. sharfuddin chowdhuryModern management of dvt dr. sharfuddin chowdhury
Modern management of dvt dr. sharfuddin chowdhuryShakila Rifat
 
Dvt Deep Venous Thrombosis
Dvt Deep Venous ThrombosisDvt Deep Venous Thrombosis
Dvt Deep Venous ThrombosisFazal Hussain
 
Deep vein thrombosis
Deep vein thrombosisDeep vein thrombosis
Deep vein thrombosisNawin Kumar
 
Doppler ultrasound in deep vein thrombosis
Doppler ultrasound in deep vein thrombosisDoppler ultrasound in deep vein thrombosis
Doppler ultrasound in deep vein thrombosisSamir Haffar
 

Viewers also liked (14)

Heamat ology taza
Heamat ology tazaHeamat ology taza
Heamat ology taza
 
N.couse case study_hit
N.couse case study_hitN.couse case study_hit
N.couse case study_hit
 
VTE & Duration of Anticoagulation
VTE & Duration of AnticoagulationVTE & Duration of Anticoagulation
VTE & Duration of Anticoagulation
 
Dvt prophylaxis
Dvt prophylaxisDvt prophylaxis
Dvt prophylaxis
 
TARTING YOUR VENOUS ACCESS PROGRAM
TARTING YOUR VENOUS ACCESS PROGRAMTARTING YOUR VENOUS ACCESS PROGRAM
TARTING YOUR VENOUS ACCESS PROGRAM
 
Lymphoproliferative disorders
Lymphoproliferative disordersLymphoproliferative disorders
Lymphoproliferative disorders
 
Iliofemoral DVT thrombolysis
Iliofemoral DVT thrombolysisIliofemoral DVT thrombolysis
Iliofemoral DVT thrombolysis
 
Ultrasound Assessment Of Chronic Venous Disease
Ultrasound Assessment Of Chronic Venous DiseaseUltrasound Assessment Of Chronic Venous Disease
Ultrasound Assessment Of Chronic Venous Disease
 
Modern management of dvt dr. sharfuddin chowdhury
Modern management of dvt dr. sharfuddin chowdhuryModern management of dvt dr. sharfuddin chowdhury
Modern management of dvt dr. sharfuddin chowdhury
 
Dvt Deep Venous Thrombosis
Dvt Deep Venous ThrombosisDvt Deep Venous Thrombosis
Dvt Deep Venous Thrombosis
 
Deep vein thrombosis
Deep vein thrombosisDeep vein thrombosis
Deep vein thrombosis
 
Dvt
DvtDvt
Dvt
 
Pathology of Skin - Common Disorders
Pathology of Skin - Common DisordersPathology of Skin - Common Disorders
Pathology of Skin - Common Disorders
 
Doppler ultrasound in deep vein thrombosis
Doppler ultrasound in deep vein thrombosisDoppler ultrasound in deep vein thrombosis
Doppler ultrasound in deep vein thrombosis
 

Similar to Hematology

Surgion sweming in blood pool Central Hospital
Surgion sweming in blood pool   Central HospitalSurgion sweming in blood pool   Central Hospital
Surgion sweming in blood pool Central HospitalHusni Ajaj
 
Treatment of venous thrombosis and pulmonary embolism
Treatment of venous thrombosis and pulmonary embolism Treatment of venous thrombosis and pulmonary embolism
Treatment of venous thrombosis and pulmonary embolism Mahmoud Elhusseiny Abolmagd
 
Coagulants and anticoagulants.pptx
Coagulants and anticoagulants.pptxCoagulants and anticoagulants.pptx
Coagulants and anticoagulants.pptxKarthiga M
 
Warfarin presentation Hani alghamdi
Warfarin presentation Hani alghamdiWarfarin presentation Hani alghamdi
Warfarin presentation Hani alghamdiHaniAlghamdi16
 
Bleeding disorders.pdf
Bleeding disorders.pdfBleeding disorders.pdf
Bleeding disorders.pdfUVAS
 
Anticoagulants d
Anticoagulants dAnticoagulants d
Anticoagulants dSara Saber
 
anticoagulants acs (2) (1).pptx
anticoagulants acs (2) (1).pptxanticoagulants acs (2) (1).pptx
anticoagulants acs (2) (1).pptxsumiaru
 
Management of deep vein thrombosis and pulmonary embolism
Management of deep vein thrombosis and pulmonary embolismManagement of deep vein thrombosis and pulmonary embolism
Management of deep vein thrombosis and pulmonary embolismsunil kumar daha
 
Anticoagulant in surgery
Anticoagulant in surgeryAnticoagulant in surgery
Anticoagulant in surgeryTenzin yoezer
 
Pulmonary thromboembolism Management and prophylaxis
Pulmonary thromboembolism Management and prophylaxisPulmonary thromboembolism Management and prophylaxis
Pulmonary thromboembolism Management and prophylaxisMd Shahid Iqubal
 
Warfarin toxicity
Warfarin toxicity Warfarin toxicity
Warfarin toxicity Amira Badr
 
Homestasis Surgery
Homestasis SurgeryHomestasis Surgery
Homestasis Surgeryaxix
 
Managing the heart value
Managing the heart valueManaging the heart value
Managing the heart valueNarayana Health
 
Oral Surgery in Patients on Anticoagulant Therapy
Oral Surgery in Patients on Anticoagulant TherapyOral Surgery in Patients on Anticoagulant Therapy
Oral Surgery in Patients on Anticoagulant TherapyVarun Mittal
 

Similar to Hematology (20)

Surgion sweming in blood pool Central Hospital
Surgion sweming in blood pool   Central HospitalSurgion sweming in blood pool   Central Hospital
Surgion sweming in blood pool Central Hospital
 
Treatment of venous thrombosis and pulmonary embolism
Treatment of venous thrombosis and pulmonary embolism Treatment of venous thrombosis and pulmonary embolism
Treatment of venous thrombosis and pulmonary embolism
 
Coagulants and anticoagulants.pptx
Coagulants and anticoagulants.pptxCoagulants and anticoagulants.pptx
Coagulants and anticoagulants.pptx
 
Warfarin presentation Hani alghamdi
Warfarin presentation Hani alghamdiWarfarin presentation Hani alghamdi
Warfarin presentation Hani alghamdi
 
Bleeding disorders.pdf
Bleeding disorders.pdfBleeding disorders.pdf
Bleeding disorders.pdf
 
Anticoagulants d
Anticoagulants dAnticoagulants d
Anticoagulants d
 
Anticoagulants
AnticoagulantsAnticoagulants
Anticoagulants
 
anticoagulants acs (2) (1).pptx
anticoagulants acs (2) (1).pptxanticoagulants acs (2) (1).pptx
anticoagulants acs (2) (1).pptx
 
Management of deep vein thrombosis and pulmonary embolism
Management of deep vein thrombosis and pulmonary embolismManagement of deep vein thrombosis and pulmonary embolism
Management of deep vein thrombosis and pulmonary embolism
 
Anticoagulant in surgery
Anticoagulant in surgeryAnticoagulant in surgery
Anticoagulant in surgery
 
Anticoagulants Nursing esther
Anticoagulants Nursing estherAnticoagulants Nursing esther
Anticoagulants Nursing esther
 
Ambulatory care2
Ambulatory care2Ambulatory care2
Ambulatory care2
 
Pulmonary thromboembolism Management and prophylaxis
Pulmonary thromboembolism Management and prophylaxisPulmonary thromboembolism Management and prophylaxis
Pulmonary thromboembolism Management and prophylaxis
 
Warfarin toxicity
Warfarin toxicity Warfarin toxicity
Warfarin toxicity
 
12anticoagulants
12anticoagulants12anticoagulants
12anticoagulants
 
DVT
DVTDVT
DVT
 
Homestasis Surgery
Homestasis SurgeryHomestasis Surgery
Homestasis Surgery
 
Managing the heart value
Managing the heart valueManaging the heart value
Managing the heart value
 
Anticoagulants
AnticoagulantsAnticoagulants
Anticoagulants
 
Oral Surgery in Patients on Anticoagulant Therapy
Oral Surgery in Patients on Anticoagulant TherapyOral Surgery in Patients on Anticoagulant Therapy
Oral Surgery in Patients on Anticoagulant Therapy
 

More from S Mukesh Kumar (15)

Pulmonology
PulmonologyPulmonology
Pulmonology
 
Psychiatry ppt
Psychiatry pptPsychiatry ppt
Psychiatry ppt
 
Preventive medicine
Preventive medicinePreventive medicine
Preventive medicine
 
Peds, surg
Peds, surgPeds, surg
Peds, surg
 
Oncology step3
Oncology step3Oncology step3
Oncology step3
 
Ob & gyn
Ob & gynOb & gyn
Ob & gyn
 
Neurology[1]
Neurology[1]Neurology[1]
Neurology[1]
 
Nephrology e tutoring2
Nephrology e tutoring2Nephrology e tutoring2
Nephrology e tutoring2
 
Infectious disease
Infectious diseaseInfectious disease
Infectious disease
 
Giant cell chest conference
Giant cell chest conferenceGiant cell chest conference
Giant cell chest conference
 
Gastroenterology
GastroenterologyGastroenterology
Gastroenterology
 
Ethics in medicine
Ethics in medicineEthics in medicine
Ethics in medicine
 
Endocrinology
EndocrinologyEndocrinology
Endocrinology
 
Dermatology
DermatologyDermatology
Dermatology
 
Rheumatology 2
Rheumatology 2Rheumatology 2
Rheumatology 2
 

Recently uploaded

ENGLISH 7_Q4_LESSON 2_ Employing a Variety of Strategies for Effective Interp...
ENGLISH 7_Q4_LESSON 2_ Employing a Variety of Strategies for Effective Interp...ENGLISH 7_Q4_LESSON 2_ Employing a Variety of Strategies for Effective Interp...
ENGLISH 7_Q4_LESSON 2_ Employing a Variety of Strategies for Effective Interp...JhezDiaz1
 
Full Stack Web Development Course for Beginners
Full Stack Web Development Course  for BeginnersFull Stack Web Development Course  for Beginners
Full Stack Web Development Course for BeginnersSabitha Banu
 
Inclusivity Essentials_ Creating Accessible Websites for Nonprofits .pdf
Inclusivity Essentials_ Creating Accessible Websites for Nonprofits .pdfInclusivity Essentials_ Creating Accessible Websites for Nonprofits .pdf
Inclusivity Essentials_ Creating Accessible Websites for Nonprofits .pdfTechSoup
 
ENGLISH6-Q4-W3.pptxqurter our high choom
ENGLISH6-Q4-W3.pptxqurter our high choomENGLISH6-Q4-W3.pptxqurter our high choom
ENGLISH6-Q4-W3.pptxqurter our high choomnelietumpap1
 
How to do quick user assign in kanban in Odoo 17 ERP
How to do quick user assign in kanban in Odoo 17 ERPHow to do quick user assign in kanban in Odoo 17 ERP
How to do quick user assign in kanban in Odoo 17 ERPCeline George
 
Karra SKD Conference Presentation Revised.pptx
Karra SKD Conference Presentation Revised.pptxKarra SKD Conference Presentation Revised.pptx
Karra SKD Conference Presentation Revised.pptxAshokKarra1
 
Like-prefer-love -hate+verb+ing & silent letters & citizenship text.pdf
Like-prefer-love -hate+verb+ing & silent letters & citizenship text.pdfLike-prefer-love -hate+verb+ing & silent letters & citizenship text.pdf
Like-prefer-love -hate+verb+ing & silent letters & citizenship text.pdfMr Bounab Samir
 
ECONOMIC CONTEXT - LONG FORM TV DRAMA - PPT
ECONOMIC CONTEXT - LONG FORM TV DRAMA - PPTECONOMIC CONTEXT - LONG FORM TV DRAMA - PPT
ECONOMIC CONTEXT - LONG FORM TV DRAMA - PPTiammrhaywood
 
USPS® Forced Meter Migration - How to Know if Your Postage Meter Will Soon be...
USPS® Forced Meter Migration - How to Know if Your Postage Meter Will Soon be...USPS® Forced Meter Migration - How to Know if Your Postage Meter Will Soon be...
USPS® Forced Meter Migration - How to Know if Your Postage Meter Will Soon be...Postal Advocate Inc.
 
INTRODUCTION TO CATHOLIC CHRISTOLOGY.pptx
INTRODUCTION TO CATHOLIC CHRISTOLOGY.pptxINTRODUCTION TO CATHOLIC CHRISTOLOGY.pptx
INTRODUCTION TO CATHOLIC CHRISTOLOGY.pptxHumphrey A Beña
 
Computed Fields and api Depends in the Odoo 17
Computed Fields and api Depends in the Odoo 17Computed Fields and api Depends in the Odoo 17
Computed Fields and api Depends in the Odoo 17Celine George
 
call girls in Kamla Market (DELHI) 🔝 >༒9953330565🔝 genuine Escort Service 🔝✔️✔️
call girls in Kamla Market (DELHI) 🔝 >༒9953330565🔝 genuine Escort Service 🔝✔️✔️call girls in Kamla Market (DELHI) 🔝 >༒9953330565🔝 genuine Escort Service 🔝✔️✔️
call girls in Kamla Market (DELHI) 🔝 >༒9953330565🔝 genuine Escort Service 🔝✔️✔️9953056974 Low Rate Call Girls In Saket, Delhi NCR
 
ECONOMIC CONTEXT - PAPER 1 Q3: NEWSPAPERS.pptx
ECONOMIC CONTEXT - PAPER 1 Q3: NEWSPAPERS.pptxECONOMIC CONTEXT - PAPER 1 Q3: NEWSPAPERS.pptx
ECONOMIC CONTEXT - PAPER 1 Q3: NEWSPAPERS.pptxiammrhaywood
 
GRADE 4 - SUMMATIVE TEST QUARTER 4 ALL SUBJECTS
GRADE 4 - SUMMATIVE TEST QUARTER 4 ALL SUBJECTSGRADE 4 - SUMMATIVE TEST QUARTER 4 ALL SUBJECTS
GRADE 4 - SUMMATIVE TEST QUARTER 4 ALL SUBJECTSJoshuaGantuangco2
 
Judging the Relevance and worth of ideas part 2.pptx
Judging the Relevance  and worth of ideas part 2.pptxJudging the Relevance  and worth of ideas part 2.pptx
Judging the Relevance and worth of ideas part 2.pptxSherlyMaeNeri
 
4.18.24 Movement Legacies, Reflection, and Review.pptx
4.18.24 Movement Legacies, Reflection, and Review.pptx4.18.24 Movement Legacies, Reflection, and Review.pptx
4.18.24 Movement Legacies, Reflection, and Review.pptxmary850239
 
ANG SEKTOR NG agrikultura.pptx QUARTER 4
ANG SEKTOR NG agrikultura.pptx QUARTER 4ANG SEKTOR NG agrikultura.pptx QUARTER 4
ANG SEKTOR NG agrikultura.pptx QUARTER 4MiaBumagat1
 
ISYU TUNGKOL SA SEKSWLADIDA (ISSUE ABOUT SEXUALITY
ISYU TUNGKOL SA SEKSWLADIDA (ISSUE ABOUT SEXUALITYISYU TUNGKOL SA SEKSWLADIDA (ISSUE ABOUT SEXUALITY
ISYU TUNGKOL SA SEKSWLADIDA (ISSUE ABOUT SEXUALITYKayeClaireEstoconing
 

Recently uploaded (20)

ENGLISH 7_Q4_LESSON 2_ Employing a Variety of Strategies for Effective Interp...
ENGLISH 7_Q4_LESSON 2_ Employing a Variety of Strategies for Effective Interp...ENGLISH 7_Q4_LESSON 2_ Employing a Variety of Strategies for Effective Interp...
ENGLISH 7_Q4_LESSON 2_ Employing a Variety of Strategies for Effective Interp...
 
Full Stack Web Development Course for Beginners
Full Stack Web Development Course  for BeginnersFull Stack Web Development Course  for Beginners
Full Stack Web Development Course for Beginners
 
Inclusivity Essentials_ Creating Accessible Websites for Nonprofits .pdf
Inclusivity Essentials_ Creating Accessible Websites for Nonprofits .pdfInclusivity Essentials_ Creating Accessible Websites for Nonprofits .pdf
Inclusivity Essentials_ Creating Accessible Websites for Nonprofits .pdf
 
ENGLISH6-Q4-W3.pptxqurter our high choom
ENGLISH6-Q4-W3.pptxqurter our high choomENGLISH6-Q4-W3.pptxqurter our high choom
ENGLISH6-Q4-W3.pptxqurter our high choom
 
How to do quick user assign in kanban in Odoo 17 ERP
How to do quick user assign in kanban in Odoo 17 ERPHow to do quick user assign in kanban in Odoo 17 ERP
How to do quick user assign in kanban in Odoo 17 ERP
 
Karra SKD Conference Presentation Revised.pptx
Karra SKD Conference Presentation Revised.pptxKarra SKD Conference Presentation Revised.pptx
Karra SKD Conference Presentation Revised.pptx
 
Like-prefer-love -hate+verb+ing & silent letters & citizenship text.pdf
Like-prefer-love -hate+verb+ing & silent letters & citizenship text.pdfLike-prefer-love -hate+verb+ing & silent letters & citizenship text.pdf
Like-prefer-love -hate+verb+ing & silent letters & citizenship text.pdf
 
ECONOMIC CONTEXT - LONG FORM TV DRAMA - PPT
ECONOMIC CONTEXT - LONG FORM TV DRAMA - PPTECONOMIC CONTEXT - LONG FORM TV DRAMA - PPT
ECONOMIC CONTEXT - LONG FORM TV DRAMA - PPT
 
USPS® Forced Meter Migration - How to Know if Your Postage Meter Will Soon be...
USPS® Forced Meter Migration - How to Know if Your Postage Meter Will Soon be...USPS® Forced Meter Migration - How to Know if Your Postage Meter Will Soon be...
USPS® Forced Meter Migration - How to Know if Your Postage Meter Will Soon be...
 
INTRODUCTION TO CATHOLIC CHRISTOLOGY.pptx
INTRODUCTION TO CATHOLIC CHRISTOLOGY.pptxINTRODUCTION TO CATHOLIC CHRISTOLOGY.pptx
INTRODUCTION TO CATHOLIC CHRISTOLOGY.pptx
 
Computed Fields and api Depends in the Odoo 17
Computed Fields and api Depends in the Odoo 17Computed Fields and api Depends in the Odoo 17
Computed Fields and api Depends in the Odoo 17
 
call girls in Kamla Market (DELHI) 🔝 >༒9953330565🔝 genuine Escort Service 🔝✔️✔️
call girls in Kamla Market (DELHI) 🔝 >༒9953330565🔝 genuine Escort Service 🔝✔️✔️call girls in Kamla Market (DELHI) 🔝 >༒9953330565🔝 genuine Escort Service 🔝✔️✔️
call girls in Kamla Market (DELHI) 🔝 >༒9953330565🔝 genuine Escort Service 🔝✔️✔️
 
ECONOMIC CONTEXT - PAPER 1 Q3: NEWSPAPERS.pptx
ECONOMIC CONTEXT - PAPER 1 Q3: NEWSPAPERS.pptxECONOMIC CONTEXT - PAPER 1 Q3: NEWSPAPERS.pptx
ECONOMIC CONTEXT - PAPER 1 Q3: NEWSPAPERS.pptx
 
GRADE 4 - SUMMATIVE TEST QUARTER 4 ALL SUBJECTS
GRADE 4 - SUMMATIVE TEST QUARTER 4 ALL SUBJECTSGRADE 4 - SUMMATIVE TEST QUARTER 4 ALL SUBJECTS
GRADE 4 - SUMMATIVE TEST QUARTER 4 ALL SUBJECTS
 
YOUVE_GOT_EMAIL_PRELIMS_EL_DORADO_2024.pptx
YOUVE_GOT_EMAIL_PRELIMS_EL_DORADO_2024.pptxYOUVE_GOT_EMAIL_PRELIMS_EL_DORADO_2024.pptx
YOUVE_GOT_EMAIL_PRELIMS_EL_DORADO_2024.pptx
 
Judging the Relevance and worth of ideas part 2.pptx
Judging the Relevance  and worth of ideas part 2.pptxJudging the Relevance  and worth of ideas part 2.pptx
Judging the Relevance and worth of ideas part 2.pptx
 
4.18.24 Movement Legacies, Reflection, and Review.pptx
4.18.24 Movement Legacies, Reflection, and Review.pptx4.18.24 Movement Legacies, Reflection, and Review.pptx
4.18.24 Movement Legacies, Reflection, and Review.pptx
 
ANG SEKTOR NG agrikultura.pptx QUARTER 4
ANG SEKTOR NG agrikultura.pptx QUARTER 4ANG SEKTOR NG agrikultura.pptx QUARTER 4
ANG SEKTOR NG agrikultura.pptx QUARTER 4
 
ISYU TUNGKOL SA SEKSWLADIDA (ISSUE ABOUT SEXUALITY
ISYU TUNGKOL SA SEKSWLADIDA (ISSUE ABOUT SEXUALITYISYU TUNGKOL SA SEKSWLADIDA (ISSUE ABOUT SEXUALITY
ISYU TUNGKOL SA SEKSWLADIDA (ISSUE ABOUT SEXUALITY
 
TataKelola dan KamSiber Kecerdasan Buatan v022.pdf
TataKelola dan KamSiber Kecerdasan Buatan v022.pdfTataKelola dan KamSiber Kecerdasan Buatan v022.pdf
TataKelola dan KamSiber Kecerdasan Buatan v022.pdf
 

Hematology

  • 1. Hematology Archer’s Online USMLE Reviews www.CcsWorkshop.com All rights reserved
  • 2. Anticoagulants      Heparin Low Molecular Weight Heparin Warfarin Direct thrombin inhibitors – leuperidin, argatroban Factor Xa Inhibitors - fondaparinaux
  • 3. Heparins Antithrombin 3 is a naturally occuring slow inhibitor of clotting pathway. Heparin binds to AT-3 and converts it from slow to rapid inhibitor by forming a ternary complex with AT-3 and thrombin    Unfractionated Heparin : Unfractionated heparin is a heterogeneous mixture of polysaccharide chains with a mean molecular weight of 15,000 Daltons. Given I.V. Requires Inpatient Rx. Higher incidence of Throbocytopenia and osteopenia. Monitored by measuring APTT LMWH: Derived from heparin. Molecular weight-5000 D. Can be given twice daily doses on OP basis. Less incidence of throbocytopenia and osteopenia. LMWHs have more anti factor X a activity than UF heparin. LMWH activity is monitored by factor Xa activity. Heparins can cause Hyperkalemia
  • 4.
  • 5. Heparin Induced Skin Necrosis        Affects middle age women with history of thrombotic disease Characterized by the formation of one or more painful red plaques or necrotic skin lesions. 5 days or more after starting heparin treatment. Earlier in those treated previously with heparin. Some patients develop thrombocytopenia when lesion first appears, often with paradoxical thrombosis.(HIT)  Is not always associated with thrombocytopenia A rare complication of heparin characterized by immune complex formation and thrombosis. HIT Should be suspected if the patient develop skin necrosis in areas of SQ injection or IV site. Rx – stop Heparin + start Direct thrombin inhibitors or heparinoids ( Leupiridin or Argatroban )  ( i.e; treat like HIT)
  • 6. Warfarin Mechanism of action Antagonist Skin necrosis Uses INR monitoring Dealing with Supratherapeutic INR on case by case basis Dealing with Sub therapeutic INR ( APLA, OBESITY, WARFARIN RESISTANCE, INTERACTIONS, NONCOMPLIANCE) 
  • 7. WARFARIN: MECHANISM OF ACTION Vitamin K epoxide WARFARIN Vitamin K reduced Inactive factors II, VII, IX, and X Proteins S and C Active factors II, VII, IX, and X Proteins S and C Prevents the reduction of vitamin K, which is essential for activation of certain factors Has no effect on previously formed thrombus
  • 8. PLASMA HALF-LIVES OF VITAMIN KDEPENDENT PROTEINS Factor II Factor VII 72h 6h Factor IX 24h Factor X 36h Peak anticoagulant effect may be delayed by 72 to 96 hours PROTEIN S HALF LIFE : shorter than above
  • 9. Coumadin Skin Necrosis • • • Warfarin Procoagulant Effect in first few hours can cause warfarin skin necrosis due to thrombosis. Concomitant heparin use can decrease the incidence Patients developing coumadin skin necrosis should be evaluated for Protein C deficiency
  • 10.
  • 11. Rx   Stop Coumadin Use heparin until the necrotic lesions heal.
  • 13.        Warfarin has a narrow therapeutic index Therapeutic INR typically targeted at 2-3. The risk of bleeding increases significantly when the INR > 4-6. However, the absolute risk is still low at 5.5 bleeding events per 1000 per day Therefore, patients with an INR < 9 and no significant bleeding  Manage by omitting subsequent doses of warfarin, more frequent monitoring of the INR, and resumption of therapy at a lower dose when the INR is therapeutic. When rapid reversal of the INR is needed, fresh frozen plasma, prothrombin complex, or recombinant factor VIIa can be administered. Administration of coagulation factors provides only a temporary solution due to the short half-life of the provided clotting factors (3-4 hours for Factor VII), compared with a duration of action of 2 to 5 days for warfarin, as well as relative instability of clotting factors upon administration. Administration of either fresh frozen plasma or factor concentrates will decrease the PT/INR for 4 to 6 hours  So, complete return to a therapeutic INR will require supplementation with vitamin K
  • 14. Rxng Supratherapeutic INR      Treatment of a supra therapeutic INR requires a balance between reducing the risk for hemorrhage while minimizing the risk of thrombembolism. Treatment approaches are based on the current INR, presence of bleeding, and the time frame in which reversal is required. Vitamin K1 CAN BE GIVEN to reverse the anticoagulation effect of warfarin. The most appropriate dose of vitamin K1 is the one that lowers the INR to a safe level without resulting in a subtherapeutic INR. High doses of vitamin K1 are effective but may lead to warfarin resistance for a week or more, resulting in an increased risk for thromboembolism. In such cases, heparin should be given until the effects of the vitamin K1 are complete. ( note this point)
  • 15. Forms of Vitamin K    Available in subcutaneous, IV and oral forms Subcutaneous route  delayed onset and is less predictable. If rapid reversal is desired, the IV route should be utilized, as this route has the fastest onset of action. ( Historically, intravenous vitamin K1 has been associated with an increased risk of anaphylaxis. A retrospective review of anaphylactic reactions associated with IV vitamin K1 from the Mayo Clinic revealed that the risk of anaphylaxis with vitamin K1 was 3 per 10,000 doses—a rate comparable to all forms of penicillin and less than that of IV iron dextran. If is administered by the IV route, lower doses and slower infusion rates are recommended)   Unless rapid reversal of the INR is critical, oral vitamin K1 is the preferred route of administration. In the United States, oral vitamin K1 is only available as a 5 mg tablet (Mephyton®). Therefore, oral doses prescribed should reflect even divisions of 5 mg (e.g., 2.5 mg).
  • 16. Forms of Vitamin K Route Advantages Disadvantages IV • Fastest Onset of Action • Subcutaneous • Lower risk of anaphylaxis • Oral • Safest route •Low risk of anaphylaxis •No IV site needed • Must be given by slow IV infusion •Warfarin resistance Delayed onset •Unpredictable response •Least desired route Slower onset of action •Warfarin resistance
  • 17. Management of Elevated INR in Patients Receiving Vitamin K Antagonists INR Bleeding Present Rapid Reduction Required Management* <5 No No Lower dose by 10% or omit dose; resume at lower dose when INR is therapeutic i.e; 2 TO 3 5-9 No No Omit one or two doses, resume at lower dose by 10%. No Yes — high risk May give vitamin K 2.5 mg orally if at increased risk for bleeding. No Yes—surgery 2-4 mg oral vitamin K1 for reduction of INR in 24 hours; if INR still high, can repeat with 1-2 mg orally. ≥9 No No Hold dose and give vitamin K1 10 mg orally to reduce INR in 24 hours; may repeat vitamin K1as necessary. Resume at lower dose when therapeutic. Any Yes—Serious Bleeding Yes Hold dose, give vitamin K1 10 mg by slow IV infusion, along with fresh frozen plasma, prothrombin complex, or recombinant factor VIIa; vitamin K1 may be repeated every 12 hours. Any Yes—Life Threatening Yes Hold dose, give prothrombin complex along with vitamin K1 10 mg by slow IV infusion; repeat if necessary depending on INR.
  • 18.    When the INR is elevated in a patient who has been on a consistent dose of warfarin for past few weeks  Always, consider why it is elevated Antibiotics can potentate by causing Vitamin k deficiency. If drug interaction is considered  stop the other drug if possible. If stopping the interacting drug is feasible and if INR < 5, no need to reduce the warfarin dose
  • 19. Addressing Sub Therapeutic INR Approach on increasing the dose Warfarin Resistance 
  • 20. Sub-Therapeutic INR    INR< 1.5 – Increase the dose by 10 to 20%, consider extra dose and repeat INR in 4 to 8 days. 1.5 to 2.4 – Increase dose by 5 to 10%. Repeat INR in one to two weeks 2.5 to 3.5 – No change in dose. Repeat INR is the # of consecutive inrange INRs (For example, if a patient has had three consecutive in-range INR values, recheck in 3 weeks. Maximum repeatable period not longer than 4 weeks )
  • 21. Foods that can cause sub-therapeutic INR  Foods with very high vitamin K content (more than 200 mcg) — Brussel sprouts, chickpeas, collard greens, coriander, endive, kale, liver, parsley, red leaf lettuce, spinach, Swiss chard, black or green teas, turnip greens, watercress. Foods with high vitamin K content (100-200 mcg) include basil, broccoli, butterhead lettuce, canola oil, chives, coleslaw, cucumbers (with peel), green onions, mustard greens, soybean oil.
  • 22. Scenarios        DVT Pulmonary Embolism DVT prophylaxis Atrial fibrillation Prosthetic Valves Mural thrombus Acute Coronary Syndrome
  • 24. Abnormal Blood Flow Abnormal Vessel Wall Abnormal Blood The Hypercoagulable State Dr. Rudolph Virchow 1821-1902
  • 25. Deep Vein Thrombosis Clinical associations Immobility Obesity Smoking Cancer Pregnancy Estrogen therapy
  • 26. Economy class syndrome    Economy-class syndrome is a name used by the media to describe Venous Thrombo-Embolism that can occur in anyone - but has been of particular worry in air travellers. Incidence of DVT & PE increases with the flight distance Prevention tips : Drink fluids, avoid caffeine and alcohol, keep moving legs and wear graduated support stockings designed for travel.
  • 28. Proximal vs. Distal DVT      Thrombosis confined to deep calf veins is Distal DVT where as thrombosis at or above popliteal veins is Proximal DVT. A distal DVT becomes clinically important when it extends proximally Risk of PE is much higher with proximal DVT. Silent PE occurs in about 50% of pts with proximal DVT. All patients with Proximal DVT are at increased long term risk for chronic venous insufficiency. Compression ultrasound is more sensitive to proximal DVT when compared to distal DVT.
  • 29. Idiopathic DVT    A case of DVT where no evidence of underlying obvious cause such as surgery, trauma or known malignancy is present. Search for a hypercoagulable state in such conditions Hypercoagulable states : inherited or acquired
  • 31.
  • 32. Hypercoagulabilty – Whom to test?     DVT occurring in a pt < 50 yrs of age Positive Family Hx of Venous thromboembolism Recurrent episodes of unexplained Venous thromboembolism Don’t forget to include Homocysteine When homocysteine levels are elevated in the presence of factor V Leiden or the prothrombin gene G20210A mutation, risk of recurrent thrombosis appears to be increased beyond the risk associated with any one defect alone
  • 33. Hypercoagulabilty – When to test?   Heparin • Controversial AT-III (heparin vs acute event) • Most coagulation based test for APLA  Hexagonal phospholipid not affected Warfarin • Protein C and protein S • Need to wait 3 weeks before testing protein S • Most coagulation based APLA tests
  • 34. Hypercoagulable states Inherited -“ The big five” • Factor V Leiden • Prothrombin gene mutation • Protein S deficiency • Protein C deficiency • Antithrombin III deficiency
  • 35. Factor V Leiden    Most common inherited risk factor Often associated with other risk factors e.g: homocysteine Dramatic increase in risk with estrogens ( OC Pills)
  • 36. Hypercoagulability states Acquired  Cancer  Antiphospholipid antibody syndrome  Nephrotic syndrome  Inflammatory bowel disease. Etiology: inflammatory cytokines, low protein s  Homocysteine Clues to Acquired state : older age at the time of onset, refractory to Warfarin , both venous and arterial thromboembolism.
  • 37. DVT and Cancer   Clues – Bilateral DVTs, Arterial and venous thrombosis and Warfarin refractory thrombosis
  • 39. Differential Diagnosis • Cellulitis • Knee pathology i.e. ruptured synovial cyst • Calf muscle strain • Calf muscle hematoma
  • 40. INVESTIGATIONS    Venogram : Gold standard. Rarely performed due to accuracy of non invasive testing. Risks: phlebitis, hypersensitivity, nephrotoxicity, cardiotoxicity Venous duplex ultrasound : non invasive, accurate, first line study in moderate to high risk DVT. Specificity : 95%. Sensitivity 97% for proximal DVT and only 73% for distal DVT. Can identify other pathology e.g.: calf haematoma, bakers cyst, abscesses MRI : 90% sensitive and specific for acute Proximal DVT.
  • 41. D-Dimer assays    D-Dimer should not be used as a screening test. Can be used as a first line test in pts with low pretest probability for DVT. The utility of D-Dimer is mainly to reduce the number of ultrasound testings. The negative predictive value of a d-dimer assay falls as the pretest probability for DVT increases. An assay with a sensitivity of 80% has a negative predictive value (NPV) of 97.6% in a low-risk patient. However, the NPV of the same assay is only 33% in high-risk patients with a pretest probability of 90% for DVT.
  • 42. CLINICAL ( PRETEST) PROBABILITY OF DVT WELLS DVT SCORE SYSTEM ( most popular )                 Clinical Parameter Score Active cancer (treatment ongoing, or within 6 months or palliative) Paralysis or recent plaster immobilization of the lower extremities Recently bedridden for >3 d or major surgery <4 wk Localized tenderness along the distribution of the deep venous system Entire leg swelling Calf swelling >3 cm compared to the asympto - matic leg Pitting edema (greater in the symptomatic leg) Previous DVT documented Collateral superficial veins (non varicose) Alternative diagnosis (as likely or > that of DVT ) +1 +1 +1 +1 +1 +1 +1 +1 +1 - 2
  • 43. Wells DVT Clinical Score ( contd )   Score of Zero – low probability ( 0 to 13%) Score 1 – 2 probability)  - moderate probability ( 13 to 30% Score > or = 3 - high probability ( 49 to 81%probability)
  • 44. Complications - DVT     Pulmonary Embolism Post obstructive syndrome : pain and edema in the affected limb without new clot formation Chronic venous insufficiency Rarely, thrombosis is massive, causing vascular compromise of the leg due to high venous pressure (i.e., phlegmasia cerulea dolens).
  • 46. Treatment   Anticoagulation – heparin, warfarin, lmwh – overlapping treatments IVC filters – greenfield filter
  • 47. OPTIMIZING WARFARIN THERAPY    Dosage to be individualized according to patient’s INR response. Target INR -2-3 Use of large loading dose may lead to hemorrhage and other complications. Initial dose: 2-5 mg once daily Maintenance dose: 2-10 mg once daily Start heparin on day 1 and warfarin in the evening of Day1 or on Day2. Heparin is usually discontinued after 4-5 days. Before discontinuing, ensure INR is in therapeutic range for 2 consecutive days  Monitor daily until INR is in therapeutic range, then 3 times weekly for 1-2 weeks, then less often (every 4 to 6 weeks)
  • 48. CONTARINDICATIONS AND PRECAUTIONS - WARFARIN      Hypersensitivity to warfarin Condition with risk of hemorrhage Hemorrhagic tendency Protein C & S deficiency Vitamin K deficiency
  • 49. SIDE EFFECTS      Hemorrhage Skin necrosis Microembolization Teratogenecity : contraindicated in pregnancy Agranulocytosis, leukopenia, diarrhea, nausea, anorexia.
  • 50. Uncomplicated DVT - OP management Any reason for Hospitalization ? potential probm with home Rx No probm with Home Rx Hospitalize Give BID S.C LMWH + Warfarin Ongoing Evaluation Give 1st dose S.C LMWH Stat Begin Warfarin in Doses 5mg QD Assign a visiting nurse for twice daily injs ( until the pt or family member can inject ) Daily PT and CBC with platelet count. After atleast 5 days twice daily injs of LMWH, a therapeutic INR for 2 consecutive days – d/c LMWH and follow warfarin carefully as per routine for DVT. Repeat physician evaluation by 5-7, if no problem, repeat 1 wk later and as per routine for DVT.
  • 51. Reasons for hospitalization in DVT     Signs and symptoms of PE Co-existing medical illness – anemia, lung disease, previous bleeding, high risk of bleeding or clotting Depression or any other mental illness that restricts pt’s co-operation and compliance with the home Rx. Insurance problem or other logistic difficulty that limits pt’s access to home Rx, nursing care, monitoring, food & shelter
  • 53. Duration depends on risk factor for DVT      Risk of recurrence depends on type of risk factor. If 1st DVT occurred after a major risk factor, recurrence is 3% where as if it occurred after minor risk factor recurrence is 10%  So, stratify pts based on risk factor and then decide duration Major transient risk factors : Major surgery, major medical illness and leg casting. Minor transient risk factors : OC Pills, HRT High risk thrombophilias : Homzygos Prothrombin gene mutation, Homozygos Factor v leiden, antithrombin, protein c and protein s deficiencies and APLA Syndrome Low risk thrombophilias : Heterozygosity for prothrobin gene mutation and Factor V leiden
  • 54. Recommendations – Duration of Anticoagulant Rx in pts with DVT Patient Risk of recurrence Duration of characteristics (%) Therapy - In the year after discontinuation Major transient risk factor b.Minor risk factor, no thrombophilia a. Idiopathic event, no or low risk thrombophilia d.Idiopathic event, high risk thrombophilia e.More than one idiopathic event f.Cancer, other ongoing risk factor Ref: NEJM, 2004, 351 3% <10% if risk factor avoided. >10% if persistent c. <10% >10% >10% >10% 3 months 6 months Until factor resolves 6 months Indefinite Indefinite Indefinite. Consider long term Rx with LMWH in pts with cancer
  • 55. Contraindications - Anticoagulant RX Absolute contraindications:  Active bleeding  Severe bleeding diatheses or platelet count< 20,000/mm3  Neurosurgery, ocular surgery or intracranial bleeding in past 10 days Relative contraindications:  Mild/moderate bleeding diatheses or thrombocytopenia  Recent major trauma  Major abdominal surgery in past 2 days  Brain metastases  Gastro/genitourinary bleeding in past 2wks  Endocarditis  Severe hypertension at presentation ( SBP>200 and DBP>120)
  • 56. Case Study        A 79-year-old man is admitted to the medical ward 3 days status post subdural hematoma drainage, C3 cervical spine fracture, and fixation of multiple extremity fractures sustained in a motor vehicle accident. The patient is now awake and oriented to person, place, and time, but is a lower cervical spine incomplete quadriplegic. Physical examination reveals some minimal sensation in the legs, but no ability to move the extremities. There is a Foley catheter in place that is draining yellow colored urine. Doppler ultrasonography demonstrates a thrombus in the left popliteal vein. The most important next step in the management of this patient is A. daily Doppler ultrasonography of the lower extremities B. inferior vena cava filter placement C. subcutaneous heparin D. tissue plasminogen activator thrombolysis E. warfarin F. weekly ventilation/perfusion scans for a pulmonary embolus
  • 57. Case Study       A 49-year-old man comes to clinic for follow up and monitoring of his oral anticoagulation levels. The patient is postoperative day 62 from a left total knee replacement. On postoperative day number 2 he suffered a pulmonary embolism. He was placed on intravenous unfractionated heparin and then oral warfarin. He was discharged home with follow-up instructions to return to the clinic for monitoring of his prothrombin time/international normalized ratio (INR) every 3 weeks. On return to the clinic today his PT/INR is found to be 22.4/7.3. His physical examination is unremarkable. The most appropriate management at this time is to A. admit the patient to the hospital B. instruct the patient to discontinue warfarin and return in 1 week C. instruct the patient to discontinue warfarin week until his next visit in 3 weeks D. give protamine sulfate, intravenously E. give vitamin K and follow up with the patient at his next visit
  • 59. Preventing Thrombosis in Pregnancy Give prophylaxis with heparin during pregnancy and continued heparin or warfarin for 6 weeks postpartum in women with high risk of recurrent thrombosis:  Recent thromboembolism (within the previous 6 months)  An indication for lifelong anticoagulation  Previous thrombosis with a thrombophilia associated with a high risk of recurrent thrombosis (homozygotes for factor V Leiden, homozygotes for the prothrombin gene mutation, or antiphospholipid antibodies  A history of recurrent thromboembolism
  • 60.   Consider heparin prophylaxis antepartum and intrapartum, and heparin or warfarin prophylaxis postpartum in women with one previous thromboembolic event even without underlying thrombophilia, family history, or other risk factors (such as obesity, smoking, recent surgery, immobilization, advanced age, and parity). Advise pregnant women, especially those with a history of thrombosis or thrombophilia, to stop smoking
  • 61. Rxng Thrombo-embolism in Pregnancy   Avoid the use of warfarin during pregnancy because it crosses the placenta and is associated with teratogenesis and increased fetal morbidity and mortality. Treat with LMWH @ therapeutic doses.  Continue long-term prophylaxis against recurrent venous thromboembolism after the postpartum period and completion of therapy using warfarin, aiming for a target INR of 2 to 3 ( usually 6 mos if no other hx of thrombophilia is present)
  • 63. Anemias               Look At MCV  Microcytic, Macrocytic & Normocytic Iron deficiency – Colon ca scenarios, Fibroids, malabsorption syndromes (Celiac sprue), Gastric bypass surgery (Iron is mainly absorbed in the duodenum and jejunum) B12, Folic acid deficiency B12 deficiency in the elderly Sideroblastic Anemia Thalassemias Autoimmune Hemolytic Anemias – initial rx sterods, then IVIG, if unresponsive splenectomy Hereditary Spherocytosis G6PD Deficiency – sulfa, dapsone, primaquine Sickle Cell Anemia – Crisis, Aplastic crisis, Acute chest syndrome, Avascular necrosis, osteomyelitis, autospenectomy, Folate supplement importance Aplastic Anemia Anemia of Renal Disease – Give EPO if Hgb < 10 and supplement iron if Tsat < 20% Anemia of Chronic Disease Bone marrow biopsy – indications in Anemia
  • 64. Microcytic Anemias     Importance of RDW Peripheral smear – increase in central pallor Iron studies – IRON/TIBC, Ferritin Differentiate from AOCD – in AOCD, ferritin normal/ high, iron low, tibc low
  • 65. Rx – Iron Deficiency      Correct underlying cause – Menorrhagia, Malabsorption Recommend Hysterectomy for fibroids with menorrhagia in a patient who has completed her family. Oral Iron Supplements – preferred Side effects oral iron – nausea, constipation If too many side effects and severe Fe deficiency – consider Iron dextran IV
  • 66.
  • 67. Hereditary Spherocytosis     Lack of central pallor Osmotic Fragility test Rx : blood transfusions, splenectomy Watch secondary hemochromatosis – use Desferrioxamine
  • 68.              A 32-year-old man of Italian descent is evaluated for a routine preemployment physical examination. He has always been healthy, and his physical examination is normal. Laboratory Studies Hematocrit 35% Mean corpuscular volume 63 fL Leukocyte count 6800/μL Reticulocyte count 40,000/μL (0.7% of erythrocytes) Platelet count 270,000/μL Results of fecal occult blood testing are negative. Peripheral blood smear shows microcytosis and many target cells. Which of the following is the best diagnostic test to evaluate the cause of the anemia and microcytosis? ( A ) Coombs' direct antiglobulin test ( B ) Measurement of hemoglobin A2 level ( C ) Glucose-6-phosphate dehydrogenase screen ( D ) Measurement of serum iron, total iron-binding capacity, and ferritin levels
  • 69.         A 22-year-old female college student of Greek descent is evaluated because she tires easily and has had palpitations when playing tennis for the past 3 months. She reports no night sweats or weight loss, and her family history is negative for anemia. She has two healthy siblings.Physical examination is unremarkable. A complete blood count shows a hemoglobin of 9.0 g/dL with a hematocrit of 28% and an erythrocyte count of 3.7 million/μL. Erythrocyte distribution width is elevated at 17% (normal range 10.5% to 14.5%). The neutrophil, lymphocyte, and platelet counts are normal. Which of the following is the most appropriate initial step in the management of this patient? ( A ) Perform hemoglobin electrophoresis for hemoglobin A2 and F. ( B ) Perform complete blood counts for siblings. ( C ) Measure serum ferritin. ( D ) Perform ultrasonography of the spleen. ( E ) Perform Southern blot analysis
  • 70. Key Points An elevated erythrocyte distribution width is consistent with iron-deficiency anemia.  Measurement of serum ferritin can differentiate iron-deficiency anemia from the anemia of thalassemia trait. 
  • 71.
  • 72. Macrocytic Anemias       Vitamin B12 def Folic acid def Alcohol/ Zidovudine Hypothyroidism Subtle clues for Vit b12 def  ataxia, neurological symps, psychosis, dementia Pernicious anemia – screen all B12 def pts with anti-parietal cell abs. ( Rx is IM b12 for life )
  • 73. Hemolytic Anemias      Labs that suggest Hemolysis  Indirect bilirubin, LDH, Reticulocyte count, Haptoglobin Peripheral smear  help to r/o microangiopathic hemolysis (Schistocytosis), increased reticulocytes Urine Hemosiderin  elevated only in uintravascular hemolysis Direct coomb +ve  Autoimmune Hemolysis. Concomitant thrombocytopenia may suggest TTP, DIC, EVANS syndrome
  • 74.
  • 75. Microangiopathic Hemolysis    Characterized by schistocytosis – fragmented RBCs on peripheral smear Seen in TTP, DIC, HUS, HELLP syndrome, CAPS, malignant hypertension Macroangiopathic hemolysis – also can have schistocytes eg: prosthetic valves.
  • 76.
  • 77. Autoimmune Hemolytic Anemias        Direct coombs +ve Microspherocytes on peripheral smear Urine hemosiderin –ve LDH elevated, Hapto may be low Retic count increased Rx : Steroids, IVIG, Splenectomy Recognize Autoimmune hemolysis in CLL
  • 78.             A 63-year-old man with stage I chronic lymphocytic leukemia (CLL) is evaluated for increasing dyspnea on exertion that has developed over the past 2 weeks. He currently takes no medications. On physical examination, he has pale conjunctivae and scattered axillary and inguinal lymphadenopathy that are unchanged from his last examination 1 year ago. His is afebrile. Pulmonary examination is normal except for tachypnea. Laboratory Studies Hematocrit 18% Leukocyte count 12,000/μL (25% polymorphonuclear leukocytes, 75%lymphocytes) Platelet count 285,000/μL A peripheral blood smear shows spherocytes, a reticulocyte count of 10%, polychromatophilia, smudge cells, and normal-appearing lymphocytes with no schistocytes. Hematocrit 1 year ago was 46%. Which of the following is the most likely cause of this new onset of anemia? ( A ) Conversion of CLL to acute lymphoblastic leukemia ( B ) Autoimmune hemolytic anemia ( C ) Disseminated intravascular coagulation ( D ) Marrow infiltration by CLL ( E ) Conversion to a large-cell lymphoma (Richter's syndrome)
  • 79.         A 27-year-old black man is admitted to the hospital for treatment of community-acquired pneumonia, for which he received erythromycin. On the second hospital day, he is lethargic and has easy fatigability, a temperature of 38.4 °C (101.1°F), and scleral icterus. The patient had a similar episode during a childhood ear infection. His two brothers have had similar problems, though his sister has not. His hematocrit, which was 40% on admission, is now 32%. His reticulocyte count is 140,000/μL (4% of erythrocytes).Liver studies show a serum total bilirubin concentration of 5.4 mg/dL and a direct bilirubin concentration of 1.4 mg/dL. A peripheral blood smear is shown. Which of the following is the most appropriate diagnostic study at this time? ( A ) Sickling test ( B ) Glucose-6-phosphate dehydrogenase measurement ( C ) Hemoglobin electrophoresis ( D ) Erythrocyte pyruvate kinase activity measurement ( E ) No further testing
  • 80. Key Point Following an acute hemolytic event due to glucose-6-phosphate dehydrogenase deficiency (G6PD),G6PD levels will be normal. Testing should be delayed for 3 to 4 weeks.
  • 81. Anemias           A 19-year-old woman comes to the student health service complaining that since the new semester has begun, she finds herself unable to focus and concentrate as well as before. She attributes this largely to feeling fatigued. She denies any other symptoms such as sadness, sleeplessness, or loss of libido. She has no other medical issues. Her medications are only oral contraceptive pills. She has never been pregnant and denies current pregnancy. She has a history of long menses, often lasting 8 days. Laboratory studies show: Hct 31% MCV 69um3 Ferritin 10mcg/L The most appropriate next step is to A. administer iron, intravenously B. administer vitamin B12, intravenously C. advise her to take folate tablets D. change her oral contraceptive formulation to estrogen only E. tell her to take oral iron tablets
  • 82.          Anemias A 5-year-old boy is brought to the clinic for a periodic health maintenance examination. He is generally healthy, enjoys school, plays well with his siblings and with other children his age. He and his family live in a housing development down the street that was built 10 years ago. Since his mother usually works until late in the evening, he tends to spend a lot of time at a friend's apartment in an old, dilapidated housing development nearby. You notice that he has unusually pale skin and mucus membranes and so you inquire about related symptoms. The mother tells you that she has noticed that he is significantly more tired than his siblings and he has been a "bit irritable" lately but she "didn't think nothing of it." He is up-to-date on all of his immunizations. There is no family history of blood disorders, however several of his playmates "are anemic." You decide to order hemoglobin, hematocrit, and a peripheral blood smear and schedule a follow-up visit in 1 week. He returns for his next appointment and you review the results of the laboratory studies. His hemoglobin is 9.5 g/dL, hematocrit is 30%, and the peripheral blood smear shows microcytic red blood cells with basophilic stippling. The most appropriate next step is to A. administer ferrous sulfate, orally B. administer dimercaprol, orally C. administer edetate disodium, orally D. determine B12 levels E. determine blood lead levels F. obtain an abdominal radiograph G. order hemoglobin electrophoresis H. Check Erythrocyte Zinc Protoporphrin levels
  • 83.             During a routine screening for mild dementia, a 78-year-old male resident of a nursing home is found to have a low serum vitamin B12 level. He does not have fatigue, dyspnea, chest pain, dizziness, unsteadiness, or paresthesias. His medical history is significant for coronary artery disease, for which he underwent coronary stent placement 5 years ago. His medications include metoprolol and atorvastatin. Physical examination does not indicate any symptoms of vitamin B12 deficiency, such as glossitis or impaired sensation. Laboratory Studies Hematocrit 43% Mean corpuscular volume 94 fL Serum creatinine 1.4 mg/dL Serum vitamin B12 level 200 pg/mL Which of the following is the most appropriate next step in the management of this patient? ( A ) Weekly vitamin B12 injections ( B ) Intrinsic factor antibody test ( C ) Serum methylmalonic acid level ( D ) Schilling test
  • 84. Subclinical B12 Deficiency - May not have any symps - Elevation of MMA is a sensitive marker of significant deficiency - Treat it!
  • 85. Sickle Cell Anemia         Sickle cell trait/ Sickle cell anemia Sickling test : sickle cell screen C/F:Sickle cell crises Acute Chest syndrome Infections Dactylitis Aplastic Crises : folic acid deficiency vs. Parvo virus Avascular Necrosis hip  get MRI Labs : HGB electrophoresis, Peripheral smear – howell-jolley bodies, stress lymphocytosis, increased retic count, anemia, hemolysis, elevated indirect bilirubin Management : Acute pain crises  manage with IV Fluids, analgesics, septic w/u, electrolyte repletion, watch for substance abusers ( clues are labs) Recurrent Acute pain crises  more than 3-4 episodes per year, use Hydroxyurea. D/W pt benefits vs. toxicity of hydrea Acute Chest Syndrome  O2, analgesics, Exchange Transfusion
  • 86.       A 17-year-old male patient with homozygous sickle cell anemia was admitted to the hospital for hydration and analgesia of a painful crisis. On hospital day 3, his condition deteriorated, with the onset of fever, new chest pain, and dyspnea. On physical examination, he has a temperature of 38.9 °C (102 °F), pulse rate of 120/min, and respiration rate of 32/min. He is using accessory muscles of inspiration and has crackles bilaterally. Chest radiographs show new, bilateral pulmonary infiltrates and a normal-sized heart. Pulse oximetry shows an oxygen saturation rate of 80% on room air. Which of the following is the most important immediate therapeutic option? ( A ) Amoxicillin ( B ) Hydroxyurea ( C ) Diuretics and digoxin ( D ) Partial-exchange transfusion ( E ) Polyvalent pneumococcal vaccine
  • 87. Key point The acute chest syndrome is characterized by chest pain, fever, diffuse pulmonary infiltrates, and hypoxia in a patient with sickle cell anemia.  The most effective therapy for the acute chest syndrome in sickle cell anemia is partialexchange transfusion to lower the hemoglobin S. 
  • 88. Splenectomy   Prior to splenectomy Patients should be immunized with pneumococcal, HIB and meningococcal vaccines Daily oral penicillin prophylaxis for splenectomized patients.
  • 89.             A 56-year-old man is evaluated for fatigue during a routine office visit. His history is significant for diverticulosis,hypertension, and supraventricular tachycardia, for which he takes aspirin, metoprolol, and ramipril. On physical examination, he is afebrile, his blood pressure is 120/80 mm Hg, and pulse rate is 80/min. No abdominal tenderness,splenomegaly, or lymphadenopathy is noted. Laboratory Studies Hemoglobin 8 g/dL Mean corpuscular volume 76 fL Leukocyte count 11,200/μL Platelet count 847,000/μL Which of the following is the most appropriate next diagnostic test for this patient? ( A ) Fluorescence in situ hybridization analysis of blood for Philadelphia chromosome ( B ) Examination of bone marrow aspirate ( C ) Serum ferritin measurement and fecal occult blood testing ( D ) Repeated complete blood count in 2 weeks after discontinuation of all current medications ( E ) Bleeding-time measuremen
  • 90. Ans. Reactive Thrombocytosis       Iron deficiency Malignancy Major Surgery Acute blood loss Primary thrombocytosis can be caused by clonal disorders, such as chronic myelogenous leukemia (CML), polycythemia vera, and essential thrombocythemia. CML is characterized by the presence of the Philadelphiachromosome in blood and marrow myeloid cells. Patients with CML typically have a high leukocyte count, high or normal platelet and erythrocyte counts, and splenomegaly. The absence of leukocytosis and splenomegaly and thepresence of severe microcytic anemia make CML unlikely Essential thrombocythemia may be associated with thrombotic and hemorrhagic complications. Patients with essential thrombocythemia have a platelet count of greater than 600,000/μL without known causes for reactive thrombocytosis; therefore, iron deficiency precludes this diagnosis.
  • 92.       A 48-year-old female nurse is evaluated for hematuria. One week ago, she experienced dysuria, for which cephalothin was prescribed. Her history is remarkable only for an uneventful tonsillectomy as a child and removal of a benign ovarian cyst at age 36; there is no family history of bleeding. The patient does not smoke or use alcohol or recreational drugs. She has a normal diet, and her weight has been stable. She has a history of depression that has been treated with numerous antidepressants, but she is not taking any medications currently. On physical examination, she has two ecchymoses on her lower extremities. Laboratory evaluation includes a prothrombin time (PT) of 28.2 sec (INR 4.2), activated partial thromboplastin time (PTT) of 45 sec (normal < 35 sec), normal thrombin time, negative D-dimer, and normal complete blood count and platelet count. Liver enzymes are within normal limits, and serum albumin is 4.1 g/dL. An inhibitor-screen mixing study shows complete correction of the PT and PTT after mixing of patient and control plasma at a 1:1 ratio. The factor VII level is 6%, factor IX level is 10%, factor VIII level is 110%, and factor V level is 95% (reference range 60% to 150%). Which of the following is the most likely diagnosis? ( A ) Acquired factor VIII inhibitor ( B ) Lupus anticoagulant ( C ) Hemophilia B (Christmas disease) ( D ) Surreptitious warfarin ingestion
  • 93. Key Point In severe vitamin K deficiency, the prothrombin time and the Partial thromboplastin time are prolonged. In mild vitamin K deficiency (including therapeutic warfarin), only the prothrombin time is prolonged 
  • 95. Screen for VWd     Screen for VWD in women with menorrhagia and no gynecologic reason for heavy bleeding, in patients with unexplained procedural bleeding, and in individuals with a personal or family history or physical evidence of mucocutaneous bleeding. Remember VWd can be asymptomatic for a long time with no hx at all and may just present with excess bleeding during surgical procedures Conduct a first-line coagulation screening work-up, including PT, PTT, CBC/platelets, and thrombin time.  If PTT is prolonged and other initial test results are normal, suspect VWD as a possible diagnosis ( BT is prolonged too. BT is N in Hemophilia ) ( Remember d/d for isolated PTT prolongation is also thrombotic condition APLA/ LA – This manifests by thrombosis though, unlike VWd which is bleeding) Initial tests for the diagnosis or exclusion of VWD include VWF function, VWF:Ag, and factor VIII activity and should be done more than once.
  • 96. Rx - VWd   In treatment of significant bleeding events or for prevention of bleeding during invasive procedures or surgeries  use Desmopressin Remember Tachyphylaxis with desmopressin ( in case of continued bleeding after desmopressin , use Cryoprecipitate/ VWF concentrate)
  • 97. Uremic Bleeding      Bleeding time is the best predictor for tendency to bleed. Desmopressin – Rx of choice in preventing bleeding for surgical procedures or in Rx of Bleeding Conjugated Estrogens Blood transfusion to keep hct high Recombinant Factor VIIa
  • 98. Hemophilia Screen for hemophilia A and B in individuals with:  A history of bleeding  Family members known to have hemophilia or be hemophilia carriers  Unexplained aPTT prolongation ( BT normal)  DX : If APTT prolonged, do mixing study. Correction suggests Factor deficiency.  Then obtain quantitative factor VIII/ IX levels and activity.  In patients with prolonged PTT and decreased factor VIII activity who have no family hx of hemophilia a, obtain VWF level to r/o VWd Hemophilias can present with deep bleeding ( Hemarthrosis) apart from superficial bleeding ( mucosal bleeding, ecchymoses, bruises with minimal trauma) RX: factor VIII/ IX concentrates ( A – Factor VIII def, B – FACTOR ix DEF)
  • 100. Thrombocytopenias       Drug Induced : HIT, GP IIb IIIA inhibitors ( abciximab, eptifibitide), Linezolid, Clopidogrel Always r/o Pseudothrombocytopenia – this is our first step. Congenital thrombocytopenia ( Some are microthrombocytopenias) Consumption Thrombocytopenias : TTP, DIC, Massive Bleeding, Sepsis ITP ITP + AIHA ( Coombs +ve, Evans syndrome)
  • 101. Platelet transfusions - Indications     Platelets < 15k Platelets<50k + ICH/Life threatening hemorrhage Platelets > 50k  no platelets ( all we need is 50k platelets for enough clotting. So, no need of transfusions for platelets>50k) Avoid platelets in TTP/ DIC unless severe bleeding which is unlikely manifestatuion in these cases.
  • 102. Idiopathic thrombocytopenic purpura           Previous CBC is a clue Rule out other causes – drugs, liver disease, HIV, HEP-C, APLA, SLE etc HIV related ITP responds to HAART/ Zidovudine most studied. Rx – mild ITP is often associated with a good prognosis. Platelet counts >50 × 109 cells/L with no history of bleeding  careful observation with regular visits and platelet count determinations Prednisone f Plts < 30k  If there is a response with a normalization of the platelet count over 1 to 2 weeks, taper by 5 to 10 mg/week. IVIG if immediate response is required i.e; before surgical procedures, or in initial treatment for patients with severe bleeding AND also, in patients who are refractory to treatment with glucocorticoids. Remember, response short-lived. Consider RhIG as outpatient therapy: In patients who are Rh(+) and have not had a splenectomy , For patients in whom splenectomy is not feasible and along with IVIG in patients in whom steroids have failed Splenectomy if no response to steroids (persistent platelet counts <10 × 109 cells/L for 4 to 6 weeks and for patients who have had ITP for 3 months with persistent severe thrombocytopenia (platelet count <30 × 109 cells/L) despite treatment) or Relapse after tapering Steroids Can try immunosuppressants (AZA) or Rituximab if splenectomy and Steroids are unsuccessful
  • 103. HIT           Usually occurs with in 5-7 days after exposure to heparin. Immediate HIT is possible if there was prior exposure in past 3 months enabling circulating antibodies Heparin induced Anti-platelet Abs  screening test Serotonin Release Assay  confirmatory test Base your decision most on clinical probability rather than these tests. Features  deep vein thrombosis, PE, Arterial thrombosis in a setting of acute thrombocytopenia after exposure to heparin. Rx  Once HIT is suspected/ diagnosed , must be treated Rx  Argatroban, leperidin overlap with warfarin Lepiridin difficult to monitor Argatroban adds to INR. So, safely discontinued after warfarin started and INR of 4.0 is reached. Repeat INR in 6 hrs after discontinuing Argatroban to confirm therapeutic range INR ( 2 to 3)
  • 104. Case Study       A 67 y/o man in previously good health is hospitalized with 2 day hx of fever and decreased consciousness. Temp is 103 F, HR 140, BP 88/50. There is no bleeding. The hgb 12.1 gm%, wbc 29,000, platelets 20,000/ul. Which of the following should be obtained next? A) Bone marrow biopsy B) Factor VIII level C) Measurement of platelet associated IgG D) Measurement of D-Dimer and fibrinogen E) Bleeding time
  • 105. Anemias  A 21-year-old man with no significant past medical history presents to office with complaints of blood in his urine and mucosal bleeding while brushing his teeth. He denies any drug or alcohol use. He has no family history of bleeding disorders. Petechiae are noted in the oral cavity, as is dried blood in the nostrils. Laboratory studies show the following: Hematocrit 32%; white blood cell count 8,000/mm3 with 60% neutrophils; platelet count 13,000; PT 13 seconds; PTT 28 seconds; LDH 1,200 U/L; elevated indirect bilirubin. Coombs' test is positive; abdominal examination is normal; and the peripheral smear shows spherocytes. What is the most likely diagnosis? (A) Gordon's syndrome (B) Bernard-Soulier syndrome (C) Felty's syndrome (D) Thrombotic thrombocytopenic purpura (E) Evans' syndrome (F) Idiopathic thrombocytopenic purpura (ITP)
  • 106. Myeloproliferative Disorders     Polycythemia vera ( differentiate from secondary polycythemia) Essential thrombocytosis CML Myelofibrosis ( “tear drop” cells)
  • 107. Polycythemia Vera        Measure RBC mass to r/o relative polycythemia Differentiate from Secondary polycythemia – R/o hypoxia which is the most common cause. Presence of leucocytosis and thrombocytosis as well as spelenomegaly are big clues that this could be primary. EPO level JAK-2 mutation Pruritis during shower often seen in primary polycythemia due to histamine release from mast cells Bone marrow biopsy if EPO is low or normal in presence of polycythemia
  • 108. Secondary Polycythemia     R/o Relative Polycythemia secondary to hemoconcentration/ dehydration  repeat CBC after Hydration Chronic Hypoxia : COPD, high altitudes, OSA, Obesity hypoventilation, intracardiac shunts, smoking  EPO elevated/ normal Neoplasms : RCC, Pheochromocytoma, Cushings disease, cerebellar hemangioblastoma, hepatoma Endocrine : Cushings disease
  • 109.         C/F and Rx Pruritis in Polycythemia vera Ruddish complexion, erythematous rashes on abdomen, extremities, purpura. Pts have increased risk of both thrombosis and bleeding Phlebotomy to keep hct < 45% Patients with symptomatic polycythemia should undergo immediate Phlebotomy Headache, Dizziness, chestpain Blurred vision, papilledema, hypertensive emergencies are indication for immediate Phlebotomy. Begin cytoreductive therapy concomitantly with phlebotomy in cases of P.Vera (Hydroxyurea).
  • 110. Hematological Malignancies       Leukemias – ALL, AML, CML, CLL Know when to treat CLL ( Anemia, thrombocytopenia, Rx – Fludaribine, Chlorambucil) CML – Studies, Rx AML  Know M3, associations and Rx Lymphomas  Hodgkins, NHL Secondary malignancies after radiotherapy ( Hodgkins survivors )  follow up mammograms
  • 113. Multiple Myeloma D/D MGUS Solitary Plasmocytoma POEMS syndrome (peripheral neuropathy, organomegaly, endocrinopathy, monoclonal gammopathy, skin changes, as well as other paraneoplastic features and osteosclerotic lesions) Waldenström macroglobulinemia
  • 114. MGUS       M-Protein < 3gm% Bone Marrow plasma cells < 10% Normal hgb, creatinine and serum calcium Normal bone survey ( x-rays not bone scan) RX : may progress to MM. There’s no evidence that chemotherapy reduces progression – so don’t treat Observation
  • 115. Solitary plasmacytoma       Single lytic lesion in the bone No evidence of anemia, renal insufficiency or high calcium M-protein may or may not be elevated 75% progression to MM in 10 yrs No chemotherapy is recommended Rx is excision and Radiation
  • 116. Smoldering Myeloma  Characteristics • Serum M protein >3 g/dL and/or bone marrow plasma cells ≥10% • Absence of anemia, renal failure, hypercalcemia, lytic bone lesions  Disease Management • Observation with treatment beginning at disease progression • Bisphosphonates • Supportive care • Participation in clinical trial
  • 117. Symptomatic MM   Patients with symptomatic myeloma require immediate treatment. Characteristics • • •  Presence of serum/urine M protein > 3gm% Bone marrow plasmacytosis (usually >30%) SYMPTOMS+  Anemia, renal failure, hypercalcemia, or lytic bone lesions Treatment — Thalidomide + dexa, lenalidomide, Melphalan
  • 118. Waldenstroms Macroglobulinemia Incidence and clinical features     Increased IgM. IgG may be decreased 1,500 cases/year in USA Median age -, 63 yrs Presenting symptoms • • • • • • Weakness and fatigue Hemorrhagic manifestations Weight loss Neurologic symptoms Visual disturbances Raynauds phenomenon 44% 44% 23% 11% 8% 3%
  • 119. Waldenstroms Macroglobulinemia: Clinical Features  Tumor infiltration • Bone marrow • Splenomegaly • Lymphadenopathy  Circulating IgM • • • •  Hyperviscosity syndrome Cryoglobulinemia Cold agglutinin disease Bleeding disorders Tissue IgM • Neuropathy 90% 38% 30%
  • 120. Waldenstorms macroglobulinemia     Symptoms of hyperviscocity syndrome ( Headache, chestpain, sob, blurred vision, papilledema) Rx: Plasmapheresis for circulating IgM complications Alkylating-agent based therapy (50-70% response rate) • • • •  Chlorambucil and prednisone Cyclophosphamide Melphalan CHOP (Cyclophosphomide, adriamycin, vincristine, prednisone) Nucleoside analogues (80-90% response rate) - Fludarabine - 2-Chloro-deoxyadenosine (2-CdA)
  • 121. Hematological Emergencies      TTP LEUCOSTASIS ( CML, AML) Acute Tumor Lysis Syndrome HIT Hyperviscocity Syndrome
  • 123. Transfusion Reactions         Febrile non hemolytic reactions Acute hemolytic transfusion reactions Delayed hemolytic transfusion reactions Anaphylactic reactions Urticarial reactions TRALI Post transfusion purpura Graft versus host disease
  • 124. Case Study       A 43 y/o m with severe acquired aplastic anemia has not responded to immunosuppressive agents. He has a HLA identical brother who has been cleared as a donor for his planned allogeneic stem cell transplant. They are both CMV negative. Which of the following will be prevented by using irradiated cellular blood products to this patient? A) Graft versus Host disease, Transfusion related B) CMV disease C) Alloimmunization D) Hemolytic transfusion reaction E) Febrile non hemolytic transfusion reaction
  • 125.        A 57-year-old woman (gravida 5, para 5) was admitted to the hospital with a bleeding duodenal ulcer for which she was given three units of packed erythrocytes. A regimen of antacids and cimetidine was initiated, and the patient was discharged after 3 days. Two days later, she returns to the hospital because of weakness, increased malaise, and dark urine. Her hematocrit, which was 37% at discharge, is now 26%. Liver studies show a total serum bilirubin of 6.1 mg/dL, with a direct value of 2.3 mg/dL, and a lactate dehydrogenase concentration of 467 U/L. Renal function is normal. Stools and a gastric lavage are negative for blood. The peripheral blood smear shows numerous microspherocytes. Which of the following is the most appropriate next step in the management of this patient? ( A ) Corticosteroid therapy ( B ) Blood typing re-evaluation and cross-match ( C ) Osmotic fragility test ( D ) High-dose immunoglobulin infusion ( E ) Emergency surgical consultation
  • 126. Key points 1. A delayed hemolytic transfusion reaction occurs several dayS after transfusion and is characterized by an elevated serum lactate dehydrogenase level and a high indirect bilirubin level along with decreasing hematocrit and the presence of microspherocytes. 2. A repeated cross-match is likely to detect antibodies missed on the original cross-match.